STS Benchmark - Cardiac Surgery General Mgmt Flashcards

1
Q

In the penicillin-allergic patient undergoing elective coronary revascularization, what is a safe alternative that can cover MRSA?
How is it administered safely to avoid hypotension?

A

Vancomycin - provides safe and effective prophylaxis if administered slowly.

*Vancomycin was first used clinically in 1958 before the antistaphylococcal penicillins, such as methicillin, were developed. Despite introduction of the cephalosporins, vancomycin is still effective in methicillin-resistant staphylococcal infections, bacterial endocarditis in patients allergic to penicillin, and for prophylaxis of bacterial endocarditis in patients with prosthetic valves and penicillin allergy.

Its most serious side effect, hypotension, can usually be avoided if vancomycin is administered cautiously as a 0.5% solution over 30-60 minutes.

As with all perioperative antibiotic regimens, 48 hours of administration is sufficient. More prolonged administration predisposes to superinfection and provides no documented decrease in wound, lung, or urinary infections. The spectrum of vancomycin is limited to gram-positive organisms.*

How well did you know this?
1
Not at all
2
3
4
5
Perfectly
2
Q

Assuming no change in myocardial contractility, mean arterial pressure or heart rate, which of the following will increase the pulmonary capillary wedge pressure needed to maintain a cardiac index of 2.0 liters/min/m2 addition of positive end-expiratory pressure to positive pressure ventilation?

  • myocardial edema
  • removal of a functioning intra-aortic balloon pump
  • conversion from sinus to junctional rhythm
  • right atrial-pulmonary artery mechanical assist device
A

Right atrial-pulmonary artery mechanical assist device.

The determinants of left ventricular (LV) performance are heart rate, preload, afterload, and contractility. If all variables except preload are held constant, cardiac index (CI) will become dependent on left ventricular end-diastolic volume (LVEDV). Compliance relates LVEDV to wedge pressure. Positive end-expiratory pressure increases the wedge pressure required to maintain CI. Conversion from sinus to junctional rhythm removes the atrial kick, decreasing LVEDV and stroke volume and increasing wedge pressure to maintain Cl. Edema reduces compliance of the left ventricle and requires increased wedge pressure to maintain LVEDV and Cl. Pulmonary stenosis and tricuspid insufficiency increase right ventricular end-diastolic pressure and volume, pushing the interventricular septum to the left and distorting the LV. Wedge pressure must then be increased to maintain LVEDV and Cl. Right atrial-pulmonary artery mechanical bypass unloads the right ventricle and, by a mechanism opposite that above, will decrease the wedge pressure required to maintain LVEDV and CI.

How well did you know this?
1
Not at all
2
3
4
5
Perfectly
3
Q

The decline in myocardial contractility that occurs after coronary artery occlusion is due to both lack of substrates and accumulation of various metabolites.
What is the most important substrate?

A

*The most important substrate is oxygen. Within minutes intramyocardial oxygen tension falls to low levels.

The attendant decrease in myocardial contractility is also associated:
* increased rate of glycolysis
* increased cellular levels of cyclic AMP
* accelerated conversion of phosphorylase from the inactive “B” form to the more active “A” form
* ATP degradation
* decreased glycogen synthesis

Some of these effects may be especially important in regions of partial ischemia where a reduced capacity for aerobic energy production may support marginal levels of function.*

How well did you know this?
1
Not at all
2
3
4
5
Perfectly
4
Q

In an intubated, febrile multiple trauma patient, systemic therapy with what should be initiated if a CVP line tip culture shows Candida and fundoscopic examination is consistent with Candida endophthalmitis.

A

amphotericin B

Complex Intensive Care Unit patients are highly susceptible to fungal infections especially if they have been on broad spectrum antibiotics (which is the case in most patients with multi-system trauma). The question addresses the problem of when to initiate anti-fungal therapy for suspected systemic fungemia or systemic fungal infection. In general the surgical literature has been more aggressive in treating fungal infections than many of the infectious disease recommendations. Although a central venous pressure line tip which grows Candida in a febrile patient may or may not be the source of the fever, all patients who have fundoscopic evidence of Candida endophthalmitis have systemic fungemia by definition and need systemic therapy.

How well did you know this?
1
Not at all
2
3
4
5
Perfectly
5
Q

When should one start systemic antifungal therapy based on blood cultures?
What about for urine?

A

In general, one positive blood culture in a patient who has positive cultures from another source and who has a persistent fever, should be a stimulus to initiate systemic therapy. One should not wait for a second culture to begin therapy.

50,000 organisms in the urine with positive fundoscopic examination would be adequate stimulus for systemic therapy. However, merely having Candida with a Foley catheter in place does not necessarily imply that the urine is the source of the fever. 50,000 per/ml Candida in the urine without budding yeast may or may not suggest a systemic problem, although Candida is obviously present. Finally, empiric amphotericin therapy for systemic fungal infections in the absence of positive cultures is not warranted except in the immunosuppressed patient who is either status posttransplant or is on immunosuppressive chemotherapy. (Candida pneumonia is extremely rare in a non-immunologically suppressed host). Empiric anti-fungal therapy in a trauma patient is usually not warranted.

How well did you know this?
1
Not at all
2
3
4
5
Perfectly
6
Q

A 70 year old man with aortic insufficiency underwent cardiac catheterization. Ejection fraction was 50%, regurgitant fraction was 25% and end-diastolic volume was 200 ml. Heart rate was 100 beats/minute. What was his cardiac output?

A

7.5 L/min

How well did you know this?
1
Not at all
2
3
4
5
Perfectly
7
Q

Tumor necrosis factor (TNF), a low molecular weight polypeptide, is an early primary mediator of inflammation. For each of the following conditions, does TNF play a central role in the pathogenesis?

  • cachexia.
  • reperfusion injury.
  • coma.
  • septic shock.
  • transplant rejection.
A

All except coma.

Tumor necrosis factor (TNF) is a low molecular weight polypeptide derived mainly from mononuclear phagocytes (monocytes and macrophages) and T-lymphocytes. To accomplish its biological functions, TNF must combine with specific receptor molecules on target cells. These TNF receptors are present on all cells except red blood cells. Soluble TNF receptors can be detected in the body fluids (serum and urine) of patients with sepsis, cancer, and fever. When TNF binds to cell receptors, gene upregulation and production of new protein follow. This communication, known as receptor signal transduction, results in the production of other cytokines that amplify and propagate the biological effects of TNF or change the host response to TNF. TNF is an early primary mediator of inflammation and plays a central role in the pathogenesis of cachexia, septic shock, tissue injury (including ischemia/reperfusion), transplant rejection, and tumor cytolysis. Biological effects of TNF on the central nervous system are fever, hypothalamic-pituitary release of corticotropin releasing factor and adrenocorticotropic hormone, anorexia, and meningeal inflammation.

How well did you know this?
1
Not at all
2
3
4
5
Perfectly
8
Q

The major physiologic mechanism that augments myocardial oxygen delivery during exercise is?

A

coronary vasodilatation

During exercise or any type of hemodynamic stress the major coronary reserve is vasodilatation. Under normal conditions, myocardium extracts 75% or more of the available oxygen in the blood, so the heart has little extraction reserve. Conversely, the normal coronary vascular resistance is three to six times the resistance of the vasculature when coronary vessels are maximally dilated, Thus, coronary arteriolar dilatation can increase myocardial oxygen delivery by up to 600%.

How well did you know this?
1
Not at all
2
3
4
5
Perfectly
9
Q

What tissue changes/effect does acute heart failure have on the following?
* ATP
* protein synthesis
* phosphocreatine
* mitochondrial mass
* lactate

A

decreased ATP levels
increased protein synthesis
decreased phosphocreatine
increased mitochondrial mass
increased lactate

The response of the heart to overload can be divided into three stages: acute short-term heart failure, compensatory hyperfunction, and cell death and fibrosis. In the first stage the reserves of the non-hypertrophied myocardium are exhausted and glycogen, ATP, and phosphocreatine levels are decreased. Lactate production (accelerated glycolysis), protein synthesis, and mitochondrial mass increase. Histologically there is edema of the ventricular wall and swelling and separation of the myofibrils. These changes lead to the second stage (i.e., hypertrophy) which is associated with the return of protein synthesis rates and RNA levels to normal. The final stage of heart failure occurs when muscle cells die and are replaced by connective tissue.

How well did you know this?
1
Not at all
2
3
4
5
Perfectly
10
Q

A 72-year-old man had a right carotid endarterectomy three days ago. Re-intubation was elected because of worsening stridor. Attempts to secure an airway by awake nasotracheal intubation after protracted use of aerosolized topical 4% lidocaine were not successful. Following cricothyroidotomy the patient was stabilized and moved to the ICU. The pulse oximeter indicates an oxygen saturation of 92% but an arterial blood gas reveals hypoxemia with a pO2 of 55 mmHg.
What is the workup and management?

A

Discordance b/w the O2 sat and pO2 should prompt a suspicion for MetHgb toxicity in the setting of topical lidocaine use. Determine if hx of G6PD defx or serotonergic drugs.
The blood gas should report methemoglobin levels.
* >5% is diagnostic.
* >30% is life threatening.
Mgmt:
* Stop offending agent.
* Support - supplement oxygen.
* If hx of G6PD def or serotonergic meds, use ascorbic acid (vit C).
* If no hx of above, use Methylene blue.
* If not working, re-evaluate and consider exchange transfusion or hyperbaric oxygen.

Iron contained in hemoglobin is usually in the ferrous form. Reduction of iron to the ferric form results in the formation of methemoglobin (MetHgb), and methemoglobinemia results from increased methemoglobin formation. Several chemicals and drugs can be responsible, including local anesthetics (cetacaine, benzocaine, or lidocaine as in this patient), nitrites and sulfonamides. Clinical effects are due to the inability of MetHgb to bind oxygen, resulting in a state of functional anemia and by an increase in the affinity of the remaining ferrous heme for oxygen. Signs and symptoms depend on the MetHb level. Levels greater than 15% are associated with cyanosis. Levels of 20-45% are associated with headache, lethargy, tachycardia, weakness and dizziness. Levels greater than 45% manifest dyspnea, acidosis, cardiac dysrhythmias, heart failure, seizures and coma. Oximetry overestimates O2 saturation when there is methemoglobinemia. CO-oximetry directly measures methemoglobin saturation and correctly yields O2 saturation. The drug of choice for treatment of methemoglobinemia is methylene blue, an electron donor that converts methemoglobin to hemoglobin. Methylene blue should be given in boluses of 1-2 mg/kg IV, anticipating an effect in 30-60 minutes. This dose can be repeated after one hour. Doses exceeding 15 mg/kg may actually cause methemoglobinemia by direct oxidation of Hgb to MetHgb. Patients with glucose 6-phosphate dehydrogenase (G6PD) deficiency will not respond to methylene blue due to decreased production of NADPH. Ascorbic acid works slowly so it is of little value. Transfusions, exchange transfusions and hyperbaric oxygen therapy are potential treatments for refractory failures of methylene blue but the other modalities have not been critically studied.

How well did you know this?
1
Not at all
2
3
4
5
Perfectly
11
Q

A 65-year-old man underwent a reoperative CABG x 4. His initial post-operative course was uneventful, but on POD# 3 thrombocytopenia was noted and examination revealed black toes and evidence of a cerebrovascular accident. An echocardiogram demonstrated thrombus in the left ventricle, and other workup confirmed the stroke and bilateral common iliac vein thromboses. HIT serologic testing was reported positive. In addition to stopping all unfractionated heparin, what else should be added?

A

an intravenous direct thrombin inhibitor

Heparin induced thrombocytopenia (HIT) was first described as an arterial thrombotic disease that is now usually called HITT (heparin induced thrombocytopenia and thrombosis). This entity was notable because the emboli were pale in color secondary to platelet domination (“white clot syndrome”). It is now recognized that thrombosis can occur in either the arterial or venous system, related to the underlying vascular damage and associated surgical risk factors. Reports suggest that HITT affects the venous system more commonly than the arterial system (4:1), but as this case illustrates, both systems can be affected simultaneously. HIT and HITT are immunological disorders in which heparin-IgG-platelet factor 4 complexes bind to platelet Fc receptors, which activate to cause aggregation and potential thrombosis. In addition to aggregation, patients may develop evidence of disseminated intravascular coagulation (DIC) during the acute thrombocytopenic episode. Evidence supporting the pro-thrombotic nature of HIT is apparent from complications that follow substitution of warfarin for discontinued heparin. This may cause an initial depletion in the level of protein C, a natural anticoagulant, and these patients can develop particularly aggressive and worsening thrombosis. Thrombin generation plays a pivotal role in the pathogenesis of HIT. Treatment must include stopping all heparin, but an intravenous direct thrombin inhibitor (lepirudin, bivalirudin, argatroban, and others) should be given. Warfarin can be started only after the platelet count has returned to normal. Thrombin inhibition is continued until warfarin is therapeutic. Utilization of a Factor 10 inhibitor (enoxaparin, other low molecular weight heparins [LMWH]) as a bridge for coumadin therapy or for prophylaxis may be efficacious, but these drugs pose a residual risk. Low molecular weight heparins may produce a positive test for HIT in patients with previous exposure or a history of HIT, and thrombocytopenia may occur. About one third of patients will demonstrate cross-reactivity to unfractionated heparin. If aggregation is demonstrated, then LMWH must not be used. Otherwise, daily platelet counts are indicated.

How well did you know this?
1
Not at all
2
3
4
5
Perfectly
12
Q

Diagnostic coronary angiography prior to elective aortic valve replacement demonstrates a small right coronary artery with a single posterior descending branch. The left anterior descending coronary artery is normal, but the circumflex coronary artery is not seen.
What are some credible explanations for this?

A
  • short left main coronary artery with selective engagement of the left anterior descending coronary artery.
  • separate origins of the left anterior descending and circumflex coronary arteries from the left aortic sinus.
  • proximal occlusion of the circumflex coronary artery from atherosclerotic disease.
  • anomalous origin of the circumflex coronary from the right coronary artery.
  • NOT likely to be congenital absence of the circumflex coronary artery.

The proximal right coronary artery is the most common anomalous origin of the circumflex coronary artery. Congenital absence of the circumflex coronary artery is extremely rare, and the lack of visualization of the circumflex on coronary angiography should prompt a thorough search for the vessel. Proximal occlusion of the circumflex coronary artery can occur, but in most instances the distal vessel will fill by collaterals from the left anterior descending (LAD) coronary artery or right coronary artery. The circumflex coronary artery may not opacify if the left main coronary artery trunk is very short and the LAD is cannulated selectively or if separate origins of the LAD and circumflex exist.

How well did you know this?
1
Not at all
2
3
4
5
Perfectly
13
Q

Under normal conditions, left ventricular subendocardial blood flow is determined primarily by?

A

Under normal conditions when net coronary perfusion pressure is adequate, regional myocardial blood flow is primarily determined by regional arteriolar resistance which, in turn, is modulated by local metabolic demands.

During normal physiology, subendocardial blood flow autoregulates and is altered minimally in response to changing diastolic perfusion intervals, aortic diastolic pressures, ventricular end-diastolic pressures, and regional diastolic intramyocardial pressures. However, with maximal vasodilatation induced by hypoxia or other stimuli, regional myocardial perfusion becomes pressure and time dependent, and hemodynamic factors assume greater importance.

How well did you know this?
1
Not at all
2
3
4
5
Perfectly
14
Q

The majority of ventricular filling occurs in which phase of the cardiac cycle?

A

The first third of diastole.

Most ventricular filling occurs during the rapid filling phase in the first third of diastole. In fact, up to 50% of total filling occurs before ventricular pressure reaches the first diastolic minimum. The combination of a high atrioventricular pressure gradient upon mitral valve opening and active restorative forces associated with myocardial relaxation probably accounts for this finding.

How well did you know this?
1
Not at all
2
3
4
5
Perfectly
15
Q

Stunned myocardium is defined as a loss of myocardial contractile force following reperfusion of ischemic myocardium that does not develop necrosis.
What happens to diastolic compliance?
What happens to oxygen utilization efficiency?

A

Myocardial contractility decreases almost immediately after ischemia begins and remains depressed far longer than the duration of the ischemic period. Fifteen to twenty minutes of ischemia depresses myocardial contractility for several days. Thirty minutes of ischemia at 37 degrees C causes loss of mitochondrial dense granules, clearing of the matrix and fragmentation of cristae; yet after 120 minutes of reperfusion, mitochondrial architecture appears normal. Reversible ischemia reduces both systolic and diastolic function. Myocardial cells develop wide I bands consistent with myocardial creep. Diastolic compliance decreases and stunned myocardium is stiff. Oxygen consumption per unit of left ventricular work increases several fold in stunned myocardium. By definition, oxygen utilization efficiency is profoundly decreased.

Although the evidence is inconclusive, recovery of oxygen utilization efficiency precedes recovery of contractile function. The depression in oxygen utilization efficiency is five to ten times longer than the period of ischemia. During reversible ischemia, permeability of the sarcoplasmic reticulum membrane increases so that excess extracellular calcium enters the myocyte during reperfusion. Cellular calcium overload results and intracellular calcium contractile functions are depressed. Reperfusion with low calcium perfusates improves contractile function of stunned myocardium. However, myocardial stunning also produces oxygen free radicals and other metabolites which may also contribute to the delayed return of contractile force.

How well did you know this?
1
Not at all
2
3
4
5
Perfectly
16
Q

What can increase the risk of protamine reaction?

A

Although not all factors are identified as significant in all studies, factors which have been shown to have a predictive influence on the risk of protamine reactions include history of fish or non-protamine allergic reaction, prior protamine exposure, and use of NPH insulin.

Protamine sulfate is a strongly cationic polypeptide that is used commonly in clinical medicine. It is administered regularly after cardiac catheterization, cardiothoracic and vascular surgical procedures, and less frequently after dialysis and leukapheresis because of its capacity to reverse the anticoagulant activity of heparin. In addition, because it delays the absorption of insulin, protamine is combined with insulin in protamine zinc insulin and neutral protamine Hagedorn insulin.
The incidence of significant protamine reactions at the end of cardiopulmonary bypass (CPB) has been reported to be as high as 10.7%. Cardiovascular effects vary in severity from mild hypotension to hemodynamic collapse, and a precipitous drop in blood pressure follows protamine administration in approximately 2% of CPB cases. Many putative risk factors for protamine reactions have arisen from anecdotal reports, and others derive from theoretical immunologic considerations.
Unfortunately, neither a positive skin test nor a positive IgE ELISA for antiprotamine antibody predicts that a patient will have a reaction. Management of protamine reactions may include volume loading, antihistamines, steroids, epinephrine or additional heparinization and reinstitution of cardiopulmonary bypass. Some additional alleged risk factors for protamine reactions include prior vasectomy, poor left ventricular function, and hemodynamic instability. A complex operation in a compromised patient that requires prolonged cardiopulmonary bypass may make separation from bypass challenging, but this is unrelated to the incidence of true protamine reactions. Alternative strategies for anticoagulation and for heparin reversal are needed.

How well did you know this?
1
Not at all
2
3
4
5
Perfectly
16
Q

What are some benefits of tight perioperative glucose control after CPB cases?

A

Studies show decreases in the prevalence of several complications including infections, prolonged inflammation, acute renal failure requiring dialysis or hemofiltration, critical illness polyneuropathy, and transfusion requirements. Patients receiving intensive insulin therapy are also less likely to require prolonged mechanical ventilation and intensive care. Attempts at tight glycemic control, however, do not seem to influence neurologic or behavioral outcomes.

Glycemic control is increasingly recognized as an important goal for perioperative patients and the critically ill. Hyperglycemia and insulin resistance are common in ICU patients, even when there is no history of diabetes. Multiple studies agree that patients who undergo cardiac surgery with hyperglycemia have increased morbidity rates (including wound and sternal infections). Mortality increases when such patients remain in the ICU more than five days. It is not clear, however, if elevated blood glucose is a marker or a pathogenic mechanism of disease in these patients. The exact mechanisms by which hyperglycemia, insulin resistance, or both contribute to complications remains under investigation. Leukocyte activity, immunoglobulin production, wound healing, collagen production, endothelial function, cardiac performance, and fluid balance are each impaired in the setting of hyperglycemia. The most recent recommendations for intensive insulin therapy include strict protocol-driven testing and management with a goal of serum glucose under 110mg/dL. Studies with a less ambitious endpoint were associated with decreases by at least half in the prevalence of several complications including infections, prolonged inflammation, acute renal failure requiring dialysis or hemofiltration, critical illness polyneuropathy, and transfusion requirements. Patients receiving intensive insulin therapy are also less likely to require prolonged mechanical ventilation and intensive care. 2009 Society of Thoracic Surgeons Recommendations are as follows: Patients with and without diabetes with persistently elevated serum glucose (180 mg/dL) should receive IV insulin infusions to maintain serum glucose 180 mg/dL for the duration of their ICU care (level of evidence A). All patients who require 3 days in the ICU because of ventilatory dependency or requiring the need for inotropes, intra-aortic balloon pump, or left ventricular assist device support, anti-arrhythmics, dialysis, or continuous veno-venous hemofiltration should have a continuous insulin infusion to keep blood glucose 150 mg/dL, regardless of diabetic status (level of evidence B). Before intravenous insulin infusions are discontinued, patients should be transitioned to a subcutaneous insulin schedule using institutional protocols (level of evidence B).

How well did you know this?
1
Not at all
2
3
4
5
Perfectly
16
Q

Discuss how protamine can have a contradictory effect on ACT levels at the end of a CPB case.

A

Excess protamine weakens clot structure and decreases platelet function. Therefore, an increased activated clotting time (ACT) after protamine reversal of heparin may be misinterpreted as residual heparin anticoagulation. Additional protamine administered to treat a prolonged ACT may further increase clotting time, reduce platelet aggregation, and potentially contribute to excess bleeding after CPB. In contrast, heparin antagonists, such as recombinant platelet factor 4 and hexadimethrine, exhibit heparin reversal activity without adversely affecting the activated clotting time

How well did you know this?
1
Not at all
2
3
4
5
Perfectly
17
Q

In patients with atherosclerotic coronary disease, myocardial blood flow at rest is?

A

The normal myocardial blood flow is approximately 90 ml per 100 gm heart muscle per minute, and values of 200 ml/100 gm/min are achieved only during extreme exercise or with maximal coronary vasodilatation. When the diameter of a coronary artery is narrowed by 50%, its luminal area is reduced by 75%, and this degree of stenosis may reduce resting blood flow depending on the coronary perfusion pressure. Myocardial “bridging” occurs during systole, and even though there is narrowing of the artery, this has little influence on coronary blood flow because most occurs during diastole. Most studies show little diminution in total left ventricular blood flow in patients with angina pectoris, whether it is stable or variant (unstable) angina. Patients with stable angina pectoris experience symptoms during exercise, and in such cases, coronary flow reserve may be inadequate to meet the myocardial demands. Resting blood flow, however, is normal.

How well did you know this?
1
Not at all
2
3
4
5
Perfectly
17
Q

A 75-year-old man is in mild respiratory distress in the intensive care unit on POD# 3 following repair of an extent I thoracoabdominal aneurysm (CT and composite shown). Proximal control of the aorta was obtained just beyond the left common carotid artery. He is quite hoarse when he tries to speak and it is difficult for him to effectively cough. When he is given water to drink he clearly aspirates and coughs violently. The most likely explanation for this man’s symptoms is?

A

left recurrent nerve injury during the aneurysm repair

In addition to the usual concerns for central nervous system injury during thoracic aneurysm repair, injury to a number of intrathoracic nerves is a function of anatomy and the nature of the patient’s pathology. The left phrenic nerve can be injured when mobilizing the aortic arch in preparation for cross-clamping, during cannulation of the superior pulmonary vein or left atrial appendage for partial left heart bypass or during division of the diaphragm for exposure of aneurysms that extend below the diaphragm. Patients with impaired pulmonary function may struggle heroically (but often unsuccessfully) from a phrenic neuropraxia. A plain chest radiograph will usually reveal a markedly elevated left hemidiaphragm. Fortunately, this complication is quite rare because the nerve is usually easy to identify as it runs beneath the mediastinal pleura and along the pericardium. Left recurrent nerve injuries are a much more common problem. They occur in approximately 10% of cases and are usually due to nerve contusion or a traction injury rather than actual nerve transection. When this occurs, patients will be hoarse and will have difficulty generating a forceful cough. The risk of aspiration is substantial. This problem is much more common in descending or thoracoabdominal aneurysm repair where control of the aorta is obtained proximal to the left subclavian artery. It may also occur in cases similar to the one described, where total arch reconstruction is performed. Fortunately, only 15% of patients will require further intervention after the diagnosis is made.

How well did you know this?
1
Not at all
2
3
4
5
Perfectly
18
Q

Preoperative infusion of magnesium sulfate in patients undergoing revascularization for unstable angina results in?

A

Increasing clinical evidence suggests that perioperative infusion of magnesium sulfate has many beneficial effects in patients who undergo revascularization for unstable angina. Patients receiving perioperative infusion of magnesium sulfate have a significantly lower incidence of postoperative ventricular arrhythmias which require treatment, a decreased incidence of postoperative hypertension which requires nitroprusside infusion, improved left ventricular stroke work index and improved cardiac index with a decreased requirement for postoperative inotropic support.

The incidence of postoperative atrial fibrillation does not appear to be influenced by magnesium treatment. The only noticeable side effect of magnesium infusion is an increased time to resumption of normal sinus rhythm after release of the aortic crossclamp. However, new conduction abnormalities such as left or right bundle branch block do not result from, and are not favorably influenced by, magnesium infusion.

How well did you know this?
1
Not at all
2
3
4
5
Perfectly
19
Q

Chronic pulmonary venous hypertension can produce miliary nodules of hemosiderin laden macrophages scattered throughout the lung parenchyma. These result from punctate hemorrhages caused by venous hypertension. Are they a recognized cause of arteritis or altered pulmonary vascular resistance?

A

No.

How well did you know this?
1
Not at all
2
3
4
5
Perfectly
20
Q

High altitude pulmonary hypertension is caused by?

A

reduced inspired oxygen tension

How well did you know this?
1
Not at all
2
3
4
5
Perfectly
21
Q

Is hypertrophy of pulmonary arteriolar smooth muscle that progresses to intimal proliferation and hyperplasia is reversible in adults who remain at altitude?

A

No

How well did you know this?
1
Not at all
2
3
4
5
Perfectly
22
Q

Angiotensin converting enzyme (ACE) inhibitors are effective in reducing pulmonary hypertension?

A

No

How well did you know this?
1
Not at all
2
3
4
5
Perfectly
23
Q

Chronic pulmonary thromboembolism usually effects medium-sized and large pulmonary arteries and can cause severe right heart failure with liver congestion (cirrhosis, ascites, and severe swelling and stasis changes of the legs).
What surgical options are there?

A

Extended pulmonary endarterectomy has been shown to be effective and specific therapy for patients with chronic thromboembolic pulmonary vascular occlusive disease. The goals of the operation are complete removal of obstructing thrombus and reactive tissue, prevention of further emboli, and avoidance of cardiopulmonary bypass if possible.

How well did you know this?
1
Not at all
2
3
4
5
Perfectly
24
Q

Persistent large left-to-right shunts increase pulmonary arterial blood flow and produce hyperkinetic pulmonary hypertension. Eventually, the patient gets arteriolar proliferation and hyperplastic changes with loss of pulmonary vascular arborization. Because of the high pulmonary vascular resistance, the shunt typically reverses to right-to-left. Patients develop cyanosis, clubbing and polycythemia.
What is the eponym for this?
Is this reversible?

A

Eisenmenger’s syndrome.

Large pulmonary vessels develop medial smooth muscle hypertrophy and increased elastic tissue which regresses if the shunt is closed. Persistence of hyperkinetic pulmonary blood flow eventually produces Eisenmenger’s disease with its arteriolar proliferation and hyperplastic changes with loss of pulmonary vascular arborization. This condition is not reversible.

How well did you know this?
1
Not at all
2
3
4
5
Perfectly
25
Q

Patients w/ L to R shunt eventually develops cyanosis, clubbing and polycythemia. Cath reveals PVR 9 Woods units.
Discuss closure of the shunt.

A

Closure of the shunt in a patient with a pulmonary vascular resistance greater than 7 or 8 Woods units (after administration of a pulmonary vasodilator) does not stop progression of the disease and such an operation is extremely risky.

How well did you know this?
1
Not at all
2
3
4
5
Perfectly
26
Q

What hemodynamic changes to the pulmonary vascular can occur with MV disease or severe L side heart failure?

A

Elevated pulmonary venous pressures due to mitral valve disease or severe left heart failure increase pulmonary arterial pressure and may over time increase pulmonary vascular resistance. Hyperplastic changes in pulmonary arterioles may develop. Reduction of pulmonary venous pressures reduces pulmonary vascular resistance acutely and chronically.

How well did you know this?
1
Not at all
2
3
4
5
Perfectly
27
Q

Is surgery for MV disease contraindicated in patients with pulmonary hypertension due to increased pulmonary venous pressure?

A

Operation is not contraindicated in patients with pulmonary hypertension due to increased pulmonary venous pressure.

How well did you know this?
1
Not at all
2
3
4
5
Perfectly
28
Q

Which valvular pathology increases risk of sudden cardiac output collapse from being an autologous blood donor? Why?

A

Autologous blood donation reduces the need for homologous blood transfusions in cardiac surgery. Blood can be collected safely from most patients in whom a temporary decrement in intravascular volume can be tolerated.

Predonation is feasible in volume loading lesions such as aortic regurgitation, left ventricular aneurysms, and mitral valve disease.

In a patient with aortic stenosis a sudden decrement in blood volume could lead to a sudden decrease in cardiac output. Autologous blood donation should be avoided for a patient with this lesion.

How well did you know this?
1
Not at all
2
3
4
5
Perfectly
29
Q

At present, the estimated risk of seroconversion after a single human immunodeficiency virus contaminated needle stick is?

A

The risk of becoming infected with human immunodeficiency virus (HIV) after a single HIV-contaminated needle stick is less than 1%. However, in surgical specialties in which there is a high risk of glove puncture (e.g., orthopaedics), even this low risk translates into a significant long-term risk. In one study, the risk to an orthopaedic surgeon operating at the San Francisco General Hospital of becoming HIV-positive over five years was estimated to be 49% (40 sticks per year of which 33 % carry the HIV virus at risk of I% with each puncture).

How well did you know this?
1
Not at all
2
3
4
5
Perfectly
30
Q

Which physical finding can differentiate severe tricuspid stenosis from severe tricuspid regurgitation?
* accenuation of the murmur during inspiration
* jaundice
* hepatomegaly
* ascites

A

Jaundice and cirrhosis do not generally occur with tricuspid regurgitation

Tricuspid regurgitation features a high-pitched, blowing, holosystolic murmur. Tricuspid stenosis typically causes a low-pitched, rumbling diastolic murmur with a presystolic crescendo if the patient is in normal sinus rhythm. However, each is best heard at the lower sternal border and each increases in intensity during inspiration due to augmented right ventricular filling.

Severe tricuspid stenosis is associated with peripheral edema, ascites, and marked hepatic congestion, often resulting in congestive hepatomegaly with cirrhosis, jaundice, malnutrition, and splenomegaly. Tricuspid regurgitation is most often functional and secondary to marked dilation of the right ventricle and tricuspid annulus.

As with tricuspid stenosis, the symptoms of tricuspid regurgitation are the consequence of systolic venous congestion, including ascites, peripheral edema, hepatomegaly, pulsations of the liver, and positive hepatojugular reflux (enhanced jugular venous pulse with liver compression). However, gross jaundice and cirrhosis do not generally occur with tricuspid regurgitation.

How well did you know this?
1
Not at all
2
3
4
5
Perfectly
31
Q

A patient with mitral stenosis has a decrease in their dyspnea and pulmonary congestion with an increase in their peripheral edema and hepatomegaly. What valvular pathology may have occurred?

A

Tricuspid stenosis rarely occurs as an isolated lesion. It is most commonly associated with mitral stenosis, and sometimes with combined mitral and aortic stenosis. Since mitral stenosis generally precedes tricuspid stenosis, most patients present initially with symptoms of pulmonary congestion. Amelioration of dyspnea and pulmonary congestion in a patient with mitral stenosis suggests the development of severe tricuspid stenosis.

How well did you know this?
1
Not at all
2
3
4
5
Perfectly
32
Q

Which cation is the major intracellular cation and which is the major extracellular cation?

A

Potassium is the major intracelluar cation while Na+ is the major extracellular cation. A 70 kg human has about 4,000 mEq of K+ of which only 60 to 70 mEq (<2%) are extracellular. A serum potassium of 3.0 reflects at least a 300 mEq loss of total body K+, as intracellular K+ moves into the extracellular space to replace ongoing K+ losses. Persistent hypokalemia leads to conservation of K+ by the kidneys and preferential excretion of H+ ion in the urine; a metabolic alkalosis results. Normal K+ losses in the urine are between 40 and 60 mEq/day.

How well did you know this?
1
Not at all
2
3
4
5
Perfectly
33
Q

What electrolyte deficiency may be most important in increasing the risk of digitalis cardiotoxicity?

A

Hypomagnesemia may be more important than hypokalemia in increasing the risk of digitalis cardiotoxicity.

Hypomagnesemia is probably the most common electrolyte abnormality found in patients hospitalized in an Intensive Care Unit (ICU). Mg++ depletion is associated with other “hypos”: hypokalemia, hypophosphatemia, hyponatremia and hypocalcemia, which are all frequently associated with the use of diuretics. The lack of magnesium in intravenous fluids and the use of aminoglycosides and diuretics (which enhance renal losses of Mg++) both contribute to hypomagnesemia. Fifty percent of body Mg++ is stored in bone and < 1% is present in plasma. Serum levels are, therefore, unreliable in the assessment of Mg++ deficiency. Low serum Mg++ is known to potentiate the cardiotoxicity of digitalis by interfering with the magnesium-dependent membrane pump that allows calcium to move into muscle fibers.

How well did you know this?
1
Not at all
2
3
4
5
Perfectly
34
Q

Normal arterial oxygen content?
Normal cardiac index?
Normal oxygen delivery (i.e., cardiac output times oxygen content in blood) is?

A

The normal arterial oxygen content (O2 bound to Hgb plus O2 dissolved in plasma: CaO2) is the product of the hemoglobin concentration (15 gm/dL) times the percent saturation (100%) times the oxygen binding capacity of hemoglobin (1.36 cc O2/gm Hgb).
When the hemoglobin concentration and body temperature are normal, the contribution to oxygen content by dissolved oxygen in the plasma is negligible.

  • Normal arterial oxygen content is ~20 cc/dL.
  • Normal cardiac index is 30 dL/min/m2 or 3 L/min/m2 (range 2.6-4.2)
  • Normal oxygen delivery is approximately 600 cc/min/m2.
How well did you know this?
1
Not at all
2
3
4
5
Perfectly
35
Q

How does a right to left intracardiac shunt affect thermodilution monitoring of cardiac output?

A

Accurate measurement of the cardiac output using thermodilution catheters depends on control of multiple variables, and it is reliable only in patients who do not have certain underlying cardiac anomalies.

For example, patients with right-to-left intracardiac shunts have loss of the indicator injectate into the left side of the circulation. Because the indicator solution never passes the thermistor, the computed cardiac output value is falsely high.

How well did you know this?
1
Not at all
2
3
4
5
Perfectly
36
Q

How does tricuspid valve regurgitation affect thermodilution monitoring of cardiac output?

A

Similar to the falsely high cardiac outputs obtained in patients with right to left intracardiac shunts - loss of the indicator injectate that never passes the thermistor.

How well did you know this?
1
Not at all
2
3
4
5
Perfectly
37
Q

How do the volume and temperature of the injectate influence the accuracy of the CO measurements in thermodilution?
What about distal migration of the catheter?

A

Falsely high cardiac output values result from using injectate that is warmer than the reference temperature, and from using less (not more) than the standard volume of injectate. In these situations, the thermistor detects less of a temperature drop and interprets this as higher flow (i.e. greater cardiac output).

Distal migration of the catheter or thermistor against the pulmonary artery wall will result in faster warming of the thermistor than is really indicated by flow of blood past the catheter tip and hence inappropriately increased cardiac output measurements.

How well did you know this?
1
Not at all
2
3
4
5
Perfectly
38
Q

A patient is suspected of having HIT. They are not bleeding, but a provider asks if they should give a platelet transfusion. What is your response?

A

Platelet transfusion should generally be avoided as bleeding is rare and platelet transfusion has been reported to precipitate thrombotic events.

Retrospective studies suggest the frequency of heparin-induced thrombocytopenia (HIT) to be 3% and of HIT with thrombosis to be 1% in patients who receive heparin for five days or more. This incidence is higher in patients following cardiopulmonary bypass. It appears that bovine and porcine heparins are equally likely to induce the development of IgG antibodies to complexes of heparin and platelet factor 4. Platelet activation releases procoagulant microparticles and local endothelial events generate thrombin. Thrombin generation helps to explain the strong association between HIT and thrombosis. In its severe form, this immune-mediated adverse drug reaction is associated with thrombosis that may result in limb amputations, stroke, graft occlusion, pulmonary embolism, phlegmasia cerulea dolens, and death. Venous thrombosis is strongly associated with HIT and is more common than arterial thrombosis.

How well did you know this?
1
Not at all
2
3
4
5
Perfectly
39
Q

Warfarin-induced venous limb gangrene, a devastating venous thrombotic syndrome, is related to what deficiency?
How is it relevant to the management of HIT?

A

Acquired protein C deficiency
Also linked with HIT - If HIT is diagnosed, consider reversal of warfarin using vitamin K.

How well did you know this?
1
Not at all
2
3
4
5
Perfectly
40
Q

Discuss the medical management, specifically anti-thrombotics and anti-platelet management in the treatment of HIT.

A

Remember that HIT is a hypercoagulable state.

UFH should be stopped.
Warfarin should be reversed with vit K.
Aspirin may be indicated to limit platelet-related thrombosis.
Danaparoid can be considered to reduce thrombin generation.
Lepirudin and argatroban can be used inhibit thrombin directly assuming an anticoagulant indication.
Theoretically, platelet ADP blockers may help (ticlopidine, clopidogrel).

Platelet transfusion should generally be avoided as bleeding is rare and platelet transfusion has been reported to precipitate thrombotic events.

How well did you know this?
1
Not at all
2
3
4
5
Perfectly
41
Q

Myocardial hypertrophy (without failure) in chronic volume overload is causally associated with what changes to the following:
* wall stress
* wall thickness
* diastolic volume
* sarcomere number and morphology

A

In chronic volume overload, such as occurs with AVR, ventricular dilatation can increase systolic wall stress via the LaPlace relationship. Wall stress is proportional to ventricular pressure and ventricular size, but inversely proportional to ventricular wall thickness. Hypertrophy, stimulated by the increased wall stress, increases wall thickness, returns the r/h ratio and wall stress to normal, reduces diastolic volumes and augments ventricular pumping function. Hypertrophy increases the number of sarcomeres in parallel and in series, and thus maintains nearly normal sarcomere morphology.

As such, hypertrophy in volume overload appears to be a physiologic adaptive mechanism that normalizes pumping function and maintains myocardial performance. In the late stage of volume overload, hypertrophy can no longer compensate for the increased work load; contractility then falls and myocardial failure ensues. It is important to recognize, however, that hypertrophy and myocardial failure are separate processes with opposite functional effects.

How well did you know this?
1
Not at all
2
3
4
5
Perfectly
42
Q

How does impaired liver function affect lactate levels?
Thiamine deficiency?
Nitroprusside use?
Anaeroic metabolism?

A

Lactate levels increase with all of these states?

Lactic acid is the end product of anaerobic metabolism and is produced by the conversion of pyruvate to lactic acid. Blood levels will increase when anaerobic metabolism occurs. Normally the liver clears lactate from the blood and impaired liver function results in increased (not decreased) blood lactic acid concentrations. Although tissue ischemia may increase blood lactate, blood flow to ischemic tissue is variable and variable amounts of lactate are washed out into the general circulation. Thus, blood lactic acid concentration is an insensitive marker of tissue ischemia. The normal homeostasis of pyruvate is altered by thiamine deficiency and cyanide (from breakdown) nitroprusside, both of which increase conversion of pyruvate to lactate.

How well did you know this?
1
Not at all
2
3
4
5
Perfectly
43
Q

Blood component transfusion therapy may be appropriate for a patient with diffuse (nonsurgical) bleeding following coronary artery bypass grafting at what level of the following labs:
* prothrombin/partial thromboplastin time (What level past control?)
* platelet
* serum fibrinogen
* bleeding time
* activated clotting time

And what will you treat with?

A
  • Fresh frozen plasma is administered when the prothrombin time and partial thromboplastin time exceed l.5 times control
  • Plt transfusions may be indicated w/ counts at 100,000/µL in the setting of diffuse bleeding.
  • Cryoprecipate therapy is indicated if hypofibrinogenemia < l00 mg/dL
  • If a qualitative platelet abnormality is demonstrated by a prolonged bleeding time (twice normal) postoperatively, then platelet transfusion is indicated
  • ACT >l50 seconds following CPB indicates circulating heparin not bound to protamine. Additional protamine is required for heparin neutralization (not component therapy).

Blood component transfusions should not be given prophylactically or in the absence of clinically significant bleeding, irrespective of laboratory values.

How well did you know this?
1
Not at all
2
3
4
5
Perfectly
44
Q

In general, what can cause PACs.

A

An anatomically dispersed but integrated system of extranodal atrial pacemakers is thought to exist with site-specific differential sensitivity to both adrenergic and cholinergic stimuli. States associated with altered sympathetic tone such as that following a myocardial infarction or following an operation may activate activity of different pacemakers within this system leading to premature atrial contractions (PACs).

How well did you know this?
1
Not at all
2
3
4
5
Perfectly
45
Q

What are some of the major characteristics of PACs on EKG?

A

1) premature occurrence of a P wave; 2) altered P wave morphology; 3) normal or increased P-R interval.

How well did you know this?
1
Not at all
2
3
4
5
Perfectly
46
Q

At the left ventricular level, what measurement is most closely related to preload at the cellular level?

A

End-diastolic volume (LVIEDV).
All pressure measurements, such as PCWP, are only estimates of true preload, which is end-diastolic fiber or sarcomere length.

How well did you know this?
1
Not at all
2
3
4
5
Perfectly
47
Q

What is compliance and how does it relate to the left ventricle?

A

Compliance is an intrinsic property of the left ventricle which determines the relation between end-diastolic volume (LVEDV) and end-diastolic pressure (LVEDP).

How well did you know this?
1
Not at all
2
3
4
5
Perfectly
48
Q

How does positive pressure ventilation affect end-diastolic volume of the left ventricle and ventricular compliance?

A

Positive pressure ventilation decreases LVEDV and compliance by increasing volume of the right ventricle, which pushes the interventricular septum to the left.

How well did you know this?
1
Not at all
2
3
4
5
Perfectly
49
Q

A 60-year-old woman in the first few hours following an uneventful mitral valve replacement has the following hemodynamic parameters: Systolic pressure 80 mmHg Heart rate 110 / min Pulmonary artery pressure 25/16 mmHg Cardiac index 2.7 L/(minm2) Systemic vascular resistance 500 dynesec/cm2.

Which pharmacologic agent is most likely to provide an optimal response ?

A

Phenylephrine.

Although the underlying cardiac pathology is not specified, left ventricular function and reserve are probably adequate since preload is not elevated. Cardiac index is satisfactory and systemic vascular resistance is low. The ideal drug for this patient would increase systemic resistance without increasing heart rate or decreasing contractility. Dobutamine is a positive inotrope that increases both contractility and heart rate. Its mild beta-2 effect reduces vascular resistance slightly. While its use in this scenario might increase the systemic pressure somewhat, it will do so at the expense of increased myocardial oxygen demand. Since the patient has no evidence of inadequate myocardial contractility, the use of this positive inotrope will provide little benefit and may adversely impact the cardiac supply/demand ratio. Epinephrine’s strong beta-1 effects will increase cardiac output by increasing contractility and heart rate. At moderate and high doses its alpha agonist effects increase systemic vascular resistance. Clearly the increased vascular resistance would benefit this patient, but epinephrine’s mixed effects will worsen tachycardia and compromise the cardiac supply/demand ratio. Milrinone, a phosphodiesterase inhibitor, reduces systemic resistance while providing a modest inotropic effect. It is contraindicated in this hypotensive patient because of an already low systemic vascular resistance. Norepinephrine would increase systemic vascular resistance, heart rate and myocardial contractility because of its strong alpha and beta adrenergic effects. In this scenario, its alpha effect would be welcome, but added myocardial oxygen consumption,aggravation of tachycardia and potential ectopy are not desired. Phenylephrine is a pure alpha agonist that will increase systemic vascular resistance. It has no direct cardiac effects and is a valuable drug when hypotension results solely from inappropriate vasodilation. The acute effects of its peripheral activity cannot be known, and other organs can be compromised by vasoconstriction. Caution is appropriate regarding increasing cardiac afterload, as left ventricular work is increased and cardiac output may fall. The need for support with alpha adrenergic agents should be somewhat unsettling for the surgeon. Continuous aggressive monitoring is mandatory and prolonged treatment with phenylephrine or metaraminol (Aramine) should be avoided.

How well did you know this?
1
Not at all
2
3
4
5
Perfectly
50
Q

Is stress ulcer ppx required for all cardiac surgery pts (every patient, every time)?

A

No.

After cardiopulmonary bypass, stress ulceration and upper GI bleeding occur at rates of 0.45% in patients without prophylaxis and 0.35% with prophylaxis. However, a definite clinical benefit has not been shown and a randomized trial in cardiac surgery has not been completed. Established risk factors include valve replacement, aortic cross-clamping, bypass time, reoperation, inflammatory states (e.g., endocarditis) and non-pulsatile flow during bypass. Enteral nutrition reduces the risk of stress ulceration, probably by maintaining gut mucosal integrity. Routine or empiric therapy is not indicated in patients unless ulceration is documented. More general studies on critically ill patients indicate a high risk for ulcers and GI bleeding, and routine stress ulcer prophylaxis is strongly indicated.

Proton pump inhibitors are at least as effective as histamine-2 antagonists at raising pH, but they are not better at reducing stress ulceration in clinical trials. In fact, all agents approved for reducing peptic ulceration are effective in critically ill patients as long as they are dosed appropriately. Several of the identified risk factors for the development of nosocomial pneumonia are similar to those for stress ulceration. Curiously, though, meta-analysis suggests that nosocomial pneumonia is the main complication of ulcer prophylaxis treatment in the ICU. Anti-acid therapy promotes gastric colonization with pathogenic bacteria, and aspiration of these bacteria can cause pneumonia.

How well did you know this?
1
Not at all
2
3
4
5
Perfectly
51
Q

In cardiac surgery patients, what risk factors create the highest risk for stress ulcers?

How can it be decreased?

A

Although there is some disagreement in the literature as to which patients are most likely to benefit from stress ulcer prophylaxis, factors that define highest risk include mechanical ventilation more than 48 hours, severe sepsis, and renal failure.

Other established risk factors include valve replacement, aortic cross-clamping, bypass time, reoperation, inflammatory states (e.g., endocarditis) and non-pulsatile flow during bypass.

Enteral nutrition reduces the risk of stress ulceration, probably by maintaining gut mucosal integrity.

How well did you know this?
1
Not at all
2
3
4
5
Perfectly
52
Q

Yesterday, a 59-year-old woman underwent left upper lobectomy for lung cancer. Today her hematocrit is 18%, but no packed red blood cells have been given because of her religious beliefs. She appears to be doing well and you plan to wean mechanical ventilation. Cardiac index is currently 5.0 L/(min*m2), oxygen saturation is 98% and inspired oxygen concentration is 40%.
Which of the following parameters is expected to be significantly altered?

Lactate
Na
pH
Mixed venous oxygen saturation (SvO2)

A

SvO2.

Decreased oxygen delivery due to anemia is compensated by several mechanisms including increased cardiac output, easier hemoglobin “downloading” due to changes in 2-3DPG levels, and an increase in the oxygen extraction ratio. Of the parameters mentioned, a decrease in the mixed venous oxygen saturation would be expected since this reflects increased oxygen extraction. Provided the patient is adequately compensated, significant anemia can be well tolerated without tissue acidosis (increased lactate) and without electrolyte abnormalities or respiratory decompensation. The optimal threshold “triggers” for transfusion remain controversial.

How well did you know this?
1
Not at all
2
3
4
5
Perfectly
53
Q

What is the O2 consumption equation?

A

O2 consumption = cardiac output * hgb (gm/dl) * 1.34 * (arterial O2 sat - mixed venous O2 sat).

How well did you know this?
1
Not at all
2
3
4
5
Perfectly
54
Q

When O2 consumption is constant and arterial blood is highly saturated, only changes in what two parameters can change mixed venous oxygen saturation?

A

cardiac output or hemoglobin concentration

How well did you know this?
1
Not at all
2
3
4
5
Perfectly
55
Q

If hemoglobin concentration is constant, how are cardiac output and mixed venous O2 sat related?

A

They are directly related. Changes in SVO2 can be used as a rough estimate of changes in cardiac output assuming the patient’s oxygen consumption isn’t changing, or they are not bleeding.

How well did you know this?
1
Not at all
2
3
4
5
Perfectly
56
Q

What increases the risk for aminoglycoside related renal failure?
How is it managed?

A

Underlying renal disease, volume depletion (though it seems to be independent of CO).

Recovery is the rule once the drug is withdrawn, but elevations in Cr may persists for as long as 10 days.

There is no specific therapy, so prevention with euvolemia and early detection with a urinalysis is best management. Furosemide will not reverse this condition (often attempted).

How well did you know this?
1
Not at all
2
3
4
5
Perfectly
57
Q

LMWH affects what?
What effect does protamine have on enoxaparin?

A

Inhibits IIa and Xa.
Protamine reverses the anti-IIa effect, but only partially the anti-Xa effect (60-80%).

How well did you know this?
1
Not at all
2
3
4
5
Perfectly
58
Q

Where does the blood flow to the AV node usually come from?
What happens with ischemia to this region?

A

The RCA usually supplies the PDA (ie is dominant), and therefore supplies the AV node.
In left dominant patients, the circumflex artery supplies the PDA.

Ischemia may cause transient or permanent complete heart block.

How well did you know this?
1
Not at all
2
3
4
5
Perfectly
59
Q

What is the preferred metabolic substrate for oxidative metabolism of the heart?

A

Long chain fatty acids.

After passive transfer from the plasma to the cytosol, fatty acids must first be activated to fatty acetyl-coenzyme A subsequent to further metabolism. Although some activated fatty acid is esterified to triglyceride, the majority serves as substrate for oxidative phosphorylation. Carnitine, a water-soluble, naturally occurring amino acid, is the requisite carrier for transport of these fatty acids from the cytosol across the mitochondrial membrane for beta oxidation. Ischemic depletion of carnitine results in the toxic accumulation of esterified fatty acid intermediates. These metabolites inhibit a variety of critical intracellular enzymes involved with regulation of cell volume, maintenance of ionic gradients, and nucleotide transfer. Fatty acid metabolites are also deleterious because they activate phospholipases and have intrinsic detergent properties that nonselectively alter subcellular membranes. Coenzyme Q plays an important role in the regulation of mitochondrial oxidative metabolism by serving as a carrier of electrons moving between the flavoproteins and cytochromes thus enhancing the flow of electrons through the mitochondrial electron transport chain. Creatine phosphate is the major transport molecule for energy transfer between the mitochondria and the cytoplasm. It receives its energy from ATP and donates its energy to ADP via a reversible reaction mediated by the enzyme creatine phosphokinase. The amino acid glutamate is an intermediary in the Krebs cycle, which after exogenous supplementation can accumulate in the cytosol . After transamination and the malate-aspartate shuttle, it is made available for mitochondrial ATP production by an oxygen-independent pathway.

How well did you know this?
1
Not at all
2
3
4
5
Perfectly
60
Q

A 65 year old man with normal preoperative left ventricular function has just returned to the intensive care unit following coronary artery bypass grafting. His intraoperative course was unremarkable. His initial hemodynamic and laboratory data includes:
Pulse: 75 beats per minute
PA pressure: 35/17 torr
Hemoglobin: 11 gm/dL
PCWP: 13 torr
Blood pressure: 130/84 torr
Cardiac index: 1.7 L/min*m2
SVR index: 4100 dynes sec cm -5/m2.

The most appropriate initial treatment of this patient is?

A

Nitroprusside.

Nitroglycerin is a peripheral venodilator and not as effective at reducing systemic arterial resistance.

The patient has a high SVR as well as HTN with a low CI. Afterload is hurting the output and must be reduced. This will reduce his myocardial oxygen consumption by decreasing wall stress in addition to increasing CI. Rewarming may help if temp is low as hypothermia can increase SVR. Pain control is also nevessary.

Volume will not help as much with an adequate PCWP.

How well did you know this?
1
Not at all
2
3
4
5
Perfectly
61
Q

What is the formula for oxygen delivery?

A

DO2 = (cardiac output) x (O2 content)

O2 content = 1.39 x Hgb x O2 saturation + (0.0031 x PaO2)

How well did you know this?
1
Not at all
2
3
4
5
Perfectly
62
Q

How do you treat decreasing mixed venous return in an intubated patient with a normal O2 sat?

What do you tell the resident who wants to increase the FiO2?

A

Treatment of decreasing mixed venous oxygen saturation must focus on increasing venous return and/or decreasing airway pressure. Crystalloid, albumin or blood transfusion may be appropriate depending on the patient’s hemoglobin level. Decreasing the tidal volume and respiratory rate will decrease airway pressure.

In the absence of hypoxia, increasing the inspired oxygen concentration will not significantly increase hemoglobin saturation or oxygen delivery. The mixed venous oxygen saturation will not be effected, and high FiO2 can contribute to oxygen toxicity.

How well did you know this?
1
Not at all
2
3
4
5
Perfectly
63
Q

Which of the agent provides chemical cardioplegic protection by cell membrane hyperpolarization?

A

ATP-sensitive potassium channel openers induce hyperpolarization by reducing the action potential duration.

How well did you know this?
1
Not at all
2
3
4
5
Perfectly
64
Q

Angiotensin converting enzyme inhibitors are established drug therapy for the treatment of depressed left ventricular function following myocardial infarction. The protective effect of these agents is mediated through?

A

afterload reduction and favorable ventricular remodeling

How well did you know this?
1
Not at all
2
3
4
5
Perfectly
65
Q

How do hyperkalemic cardioplegic solutions work?

A

With K+ concentration between 12-25 mM, they depolarize the membrane potential to -50 mV from a resting potential of -80 mV. At this depolarized potential, the fast Na+ channels are inactivated resulting in diastolic arrest.

How well did you know this?
1
Not at all
2
3
4
5
Perfectly
66
Q

Coronary bypass grafting has been very successfully performed utilizing hypothermia with fibrillatory arrest (rather than cardioplegia). What are some important aspects of this technique?

A

A number of surgeons have very successfully used hypothermia with fibrillatory arrest as their primary system of myocardial protection during coronary artery bypass. If this technique is utilized, attention to detail is necessary.
Since the heart continues to be perfused, myocardial cooling necessarily means systemic cooling. Ordinarily, systemic temperatures are reduced to 20-25 C, and the heart is allowed to fibrillate as the body is cooled. Induced electrical fibrillation of the heart is not necessary.
Active venting is always employed.
Such active venting poses a threat if air is introduced due to the development of negative intracavitary pressure. This air is difficult to remove at the end of the operation - several practitioners of this technique monitor the pressure in the LV w/ a goal of > 5 mmHg.
This technique cannot be used if AI prevents adequate decompression.
A third critical principle of this technique is maintenance of adequate coronary perfusion pressure at 80 mmHg by systemic infusion of alpha adrenergic agents.

How well did you know this?
1
Not at all
2
3
4
5
Perfectly
67
Q

What do studies show about the preoperative use of ASA before CABG, particular in regards to their effects on bleeding and transfusion requirement?

A

Of 21 studies identified on this subject, there were 6 randomized controlled trials (RCTs) that were viewed as Level A evidence. All RCTs except one found that preoperative aspirin results in either increased blood loss (measured by drainage from mediastinal tubes), increased transfusion rates, or increased frequency of re-exploration. Because of the consistent finding of aspirin-associated increased blood loss in the highest quality studies, consensus panels agree that patients who receive aspirin before operation are at increased risk for above normal postoperative bleeding and blood transfusion after CABG.

Other factos: preop plavix, vWF, preoperative anemia (especially those w/ low preop blood volume, including many women).

How well did you know this?
1
Not at all
2
3
4
5
Perfectly
68
Q

Autotransfused shed blood is “red cells suspended in serum”. What happens to the different blood components in this blood?

A

The blood is defibrinated; platelet counts are not normal but are usually between 30,000-60,000/mm3.
The hematocrit will vary with the hematocrit of the patient, ranging between 19% and 30%.
Factor VIII and XI levels are somewhat diminished but not depleted to values that would affect intrinsic coagulation.
Fibrinogen levels are very low.
Fibrinopeptide A and other peptides, generated when fibrin is lysed by plasmin or released when thrombin cleaves fibrinogen, are elevated.

How well did you know this?
1
Not at all
2
3
4
5
Perfectly
69
Q

What are the major physiologic determinants of myocardial oxygen consumption?
Discuss cardioplegia’s effect on oxygen requirement compared to hypothermia and beating, vented states.

A

Heart rate, afterload, inotropy.

A beating, vented (and bypassed) heart still requires 3.0-4.0 ml/min/100 gm heart.
Profound topical hypothermia (i.e., 12C), however, decreases oxygen needs another 50% compared to normothermia.
Decreasing temperature to 25C reduces MVO2 by 20%.

The potassium arrested heart even at normothermia only consumes 1.0-1.3 ml/min/100 gm left ventricle. This modality alone decreases MVO2 65-80% compared to the empty-beating state.

How well did you know this?
1
Not at all
2
3
4
5
Perfectly
70
Q

Reoperation for coronary disease is indicated for a 78-year-old man. What is the most sensitive surveillance modality to assess for atherosclerotic disease of the ascending aorta?

A

Epi-aortic ultrasound scanning.

TEE imaging of the distal ascending aorta and arch is poor and these are typical sites for aortic crossclamping and cannulation

How well did you know this?
1
Not at all
2
3
4
5
Perfectly
71
Q

What is the most important factor in defining stroke risk for patients who require CPB?

A

Severe atherosclerotic disease

How well did you know this?
1
Not at all
2
3
4
5
Perfectly
72
Q

Describe characteristics of warm blood cardioplegia.
Dependent on wat ion?
Reduces O2 requirement by how much?
How much O2 is delivered?

A

Warm blood cardioplegia is a myocardial protection technique that depends upon hyperkalemic myocardial arrest to reduce oxygen requirements to one-fifth of normal. Oxygen delivery is decreased as blood is delivered to the heart at approximately 150 ml per minute, but this delivery rate exceeds the greatly reduced oxygen demand.

An advantage of warm blood cardioplegia is that normothermia probably maintains the balance of biochemical processes necessary for cellular homeostasis and avoids damage to blood elements caused by hypothermia. Hypothermia may impede rapid recovery of myocardial biochemical and contractile function. For this reason, warm blood cardioplegia may be superior for the recovery of ischemic myocardium.

73
Q

Advantages of retrograde plegia?

A
  • adequate myocardial protection of the potassium-arrested heart
  • extensive coronary venous network allows uniform cardioplegic delivery
  • washout of metabolites
  • transmural cooling when antegrade infusion is technically difficult or compromised by coronary arterial obstructions
  • in severe, evolving ischemia, the coronary venous system provides rapid and direct access for arresting or restorative perfusates to reach jeopardized myocardium

retroperfusion cardioplegic techniques still require aortic crossclampin

74
Q

Why do some surgeons infuse carbond dioxide into the pericardial well?

A

CO2 is the most soluble gas of interest and can be used to displace nitrogen (not very soluble). The benefit of this is unproven.

75
Q

How do you manage cerebral edema from a large air embolus during CPB?

A

Steroids and mannitol

76
Q

What is the calcium paradox as it relates to cardiopulmonary bypass and myocardial perfusion?

A

The calcium paradox is a consequence of acalcemic myocardial perfusion. Subsequent reperfusion with calcium containing solutions leads to rapid accumulation of calcium in the myocyte. A pathologic decrease in ventricular compliance occurs.
It is generally felt that a small amount of calcium in the perfusate will prevent the occurrence of calcium paradox. Therefore, even large doses of citrate in a blood cardioplegia solution will not be associated with a calcium paradox. Calcium channel blockers do not seem to prevent the calcium paradox.

77
Q

The contact of blood with foreign surfaces of the cardiopulmonary bypass circuit triggers an intense activation of coagulation and inflammatory cascades. The role of “biocompatible” extracorporeal surfaces is to attenuate these responses to minimize “post-perfusion syndrome” and homologous blood product use.
What are examples of these advances in the CPB circuit?

A
  • heparin-bonded cardiopulmonary bypass circuits
  • large-bore directional arterial cannulas
  • centrifugal pumps
  • hollow fiber membrane oxygenators
  • closed venous reservoirs
  • minimal use of cardiotomy suckers to reduce blood-air interface
  • low prime volume to decrease dilution of clotting factors and strict transfusion protocols
78
Q

Pre-treatment with pharmacologic agents may enhance myocardial protection techniques. The propensity of the hypertensive, hypertrophied heart to develop ischemic contracture was recognized early in the evolution of open heart surgery.
What can be given pre-bypass to prevent this ischemic contracture?

A

Beta-blockade with propranolol was found to be an effective technique for prevention of ischemic contracture.

79
Q

A 49 year old renal dialysis patient is undergoing a mitral valve replacement. Ten minutes after rewarming has begun, the perfusionist reports that the mixed venous oxygen saturation is 47%. Bypass flow is 2.4 l/(min x m2), mean systemic blood pressure is 80 mmHg, arterial oxygen saturation is 100%, hematocrit is 23%, bladder temperature is 36C and the patient’s hands and feet are cold.
What can be done?

A

Deepening the level of anesthesia may dramatically decrease oxygen demand and lead to a balance between oxygen supply and demand in this situation.

Increasing the flow improve O2 supply, but would require a vasodilator to deal with the MAP of 80.

80
Q

The protective effect of cardiac hypothermia on postischemic myocardial metabolism and function is incrementally related.
How much does metabolic tissue acivity decrease (percent) for each degree celsius decrease?

A

5% decrease in tissue metabolic activity for each 1°C decrement. VanHoff’s law.

Lowering myocardial temperature not only prolongs and potentiates the effect of chemical cardioplegia, it also conserves energy in substrate stores by slowing metabolic processes. Uniform cardiac hypothermia can only be maintained by core perfusion with cold solutions in addition to copious topical irrigation and limiting the rewarming effect of the systemic perfusate. Rewarming can be limited by using systemic hypothermia, diverting blood from contacting the endocardial surfaces, and lowering systemic perfusion rates to decrease bronchial flow. These measures, besides intermittent cardioplegic infusions, can abolish or avoid regional and transmural temperature gradients. Hypothermia alone, however, is not a satisfactory means of protecting the heart and it must be combined with chemical cardioplegia.

In addition, hypothermia prevents massive intracellular accumulation of calcium upon restoration of reperfusion following normothermic ischemia. Following cold intermittent chemical cardioplegia, this calcium paradox is not a significant clinical problem. Although there is a shift of the oxyhemoglobin dissociation curve, all oxygenated solutions can deliver an oxygen supply greater than the demand of a cold heart. Even aerated crystalloid solutions can avoid an “oxygen debt.”

81
Q

What is the difference in cold and warm blood cardioplegia in terms of cell volume regulation and coagulation?

A

Cell volume regulation and coagulation are impaired in cold blood cardioplegia.

However, cold blood plegia decreases the oxygen demands of the potassium-arrested heart, acts synergistically with hyperkalemia to maintain diastolic arrest, and increases the pH of any aqueous environment. This translates into preservation of high energy phosphate stores, rapid and reliable cessation of electromechanical activity, and avoidance of acidosis.

Although there is controversy as to the superiority of continuous vs. intermittent cold cardioplegia, warm cardioplegia must be administered continuously to avoid ischemic injury, as the oxygen needs of the arrested heart at 37C are 3-4 fold greater than at 15C.

82
Q

A patient is POD2 after MVR. She is doing well and tolerating PO. Someone orders an amylase, which is elevated. There is no abdominal pain.
What is happening? Should any more workup be done?

A

Check a lipase. Even if elevated, would only trend it and monitor for symptoms.

Asymptomatic hyperamylasemia is a common occurrence after cardiac surgery, occurring in nearly 1/3 of patients. Most patients will have isolated, asymptomatic hyperamylasemia which will have no adverse sequelae. Indicators that hyperamylasemia may indicate clinically significant pancreatitis include abdominal symptoms (nausea, vomiting, abdominal pain), or associated elevated lipase levels. If the patient remains asymptomatic and lipase levels are normal, further evaluations and treatments (like nasogastric suction and abdominal computed tomography) are unnecessary and may be avoided.

83
Q

A patient is femorally cannulated for CPB. 3 minutes after institution of CPB, the perfusion line pressure increases, the radial arterial line pressure drops, and the pump reservoir is not able to fill. The femoral site looks ok, and the abdomen is not distended.
What is likely happening, and what do you do?

A

Retrograde dissection.
Stop bypass and have anesthesia support the patient.
Inspect the aorta for discoloration, and look at it under TEE to see if the dissection has propogated to the ascending aorta.
At the same time, prepare to move the arterial cannula to the ascending aorta. As long as this can be established, you can continue with the case with a plan to get a postop CT scan and treat a type B dissection.

84
Q

Retrograde cardioplegia should be placed where in relation to a 2-stage venous cannula? How do you confirm position?

A

Place it anterior/medial to the RA venous cannula. Advance it at a 45 degree angle through the coronary sinus until it meets resistance near the base of the LAA.
Position confirmation:
1. dark blood return (darker than peripheral venous blood)
2. palpation of the tip near the LAA
3. perfusion pressure at 40 mm Hg (dangerous if much higher) at a flow rate of 150-200 cc/min, <20 suggests the balloon is not inflated or not occlusive

85
Q

A symptomatic 72-year-old woman has echo findings of 3+ aortic regurgitation, 4+ mitral regurgitation, a left ventricular ejection fraction of 40% with global hypokinesis and moderate right ventricular dysfunction. Cardiac catheterization reveals no coronary artery disease. The patient is scheduled for aortic valve replacement and mitral valve repair. The best strategy for myocardial protection is?

A

Clamp, ostial antegrade, then retrograde.

Optimal perfusion of the ventricular myocardium (both left and right ventricles) is probably via antegrade flow if coronary disease is minimal. Retrograde delivery is convenient and it avoids coronary ostial trauma. However, shortcomings of retrograde cardioplegia delivery to the right ventricle are significant, and post-bypass air embolism that frequently selects the right coronary artery can compound preservation and recovery difficulties. Differential apoptosis of left and right ventricular myocytes secondary to retrograde cardioplegia delivery has been demonstrated in laboratory work. In the setting of significant (>2+) aortic regurgitation, cardioplegia administered into the aortic root is of limited utility. Retrograde cardioplegia alone will achieve diastolic arrest after aortic cross-clamp application, although more time is required than by antegrade delivery. Retrograde delivery nearly always provides adequate left ventricular protection, but right ventricular delivery is less certain. Antegrade cardioplegia, when used in this setting, should be administered directly into the coronary ostia. In the described patient with reduced left and right ventricular function, particular care must be exercised to protect the right ventricle. This is best achieved with cardioplegia administered directly into the right coronary ostium. Attention to the amount and color of blood returning from the coronary ostia determines if antegrade ostial perfusion of the left coronary is also appropriate. Therefore, an expeditious double valve operation with both antegrade and retrograde cardioplegia is optimal in this setting.

86
Q

What are the advantages of retrograde plegia in redo coronary operations?

A

Avoids distal atheroembolism from diseased but patent vein grafts, can flush back debris that occur as a result of manipulating diseased grafts, is the only way to deliver plegia to areas supplied by in-situ grafts

87
Q

How is the transport of oxygen to tissue affected by hypothermia, specifically in hemoglobin and plasma?

A

Hypothermia leads to an increase in the deliver of oxygen to the tissues by plasma. It is the opposite for hemoglobin.

Hypothermia shifts the oxyhemoglobin dissociation curve to the left as does alkalosis or a fall in carbon dioxide concentration. Oxygen is tightly bound to cold hemoglobin and is not released to the tissues. Therefore, the delivery of oxygen to the tissue by hemoglobin is decreased as the patient is cooled. The solubility of oxygen in plasma is dramatically increased as cooling occurs. This is particularly true in very cold oxygenated cardioplegia solutions. Therefore, hypothermia leads to an increase in the delivery of oxygen to the tissues by plasma.

88
Q

A 57 year old patient with severe triple vessel disease has three occluded vein grafts and a patent left internal mammary artery (LIMA) graft to a large left anterior descending (LAD) coronary artery. Intermittent antegrade/retrograde cold blood cardioplegia is planned with a systemic temperature of 280C and clamping of the LIMA graft. During placement of the retrograde coronary sinus catheter, perforation of the coronary sinus occurrs.
What do you do?

A

Cool to 22C, infuse antegrade blood cardioplegia via the aorta and down each vein graft as it is completed with the LIMA occluded and monitoring closely for cardiac activity. Repair the coronary sinus at the end of the procedure.

An attempt to repair the coronary sinus at the beginning of the procedure is unwise because any attempt to utilize the coronary sinus for further cardioplegia infusion is likely to lead to destruction of the repair and further damage. The coronary sinus should be abandoned as a route for cardioplegia delivery and should be repaired at the end of the procedure.

It is probably hazardous to allow normal blood to perfuse the left anterior descending coronary artery region while cardioplegia is administered to the remainder of the heart. It is safer to cool systemically and then occlude the left internal mammary artery. The additional cooling should lead to satisfactory protection of the left anterior descending region even if limited cardioplegia perfusion of this critical region is achieved.

89
Q

A 55 year old man with rheumatic valvular heart disease underwent an uneventful mitral valve replacement. The patient was weaned from cardiopulmonary bypass without inotropic support. Protamine administration was begun via a central venous line. The patient rapidly became hypotensive with a mean blood pressure of 40 mmHg, a central venous pressure of 3 mmHg and a pulmonary artery pressure of 15/6 mmHg. The anesthesiologist noted that the patient was developing hives.
What happened?
Appropriate initial treatment includes what?

A

Anaphylactic reaction to protamine. Remember that this also can be accompanied by bronchospasm.

Initial therapy:
* DC protamine and all anesthetic agents.
* Inspired oxygen increased to 100%.
* Resuscitation w/ volume expansion and boluses of epi to increase SVR and BP.
* Secondary treatment: corticosteriods, antihistamines, catecholamine infusion, and sodium bicarbonate.

90
Q

Following induced global ischemia, early reperfusion injury/myocardial edema can be aggravated by what?
What can reduce reperfusion injury?

A

During early reperfusion, myocardial edema can be aggravated by raising coronary artery perfusion pressure to normal or above normal levels.

Leukocyte depletion has been shown to reduce reperfusion injury as have hyperosmolar and hypocalcemic reperfusion solutions. Anti-oxidants, including mannitol, may reduce reperfusion injury by limiting exposure to oxygen free radicals.

91
Q

A predictable side effect of total circulatory arrest at profound hypothermia (15 C) for 60 minutes is

A

Total circulatory arrest with profound hypothermia is generally combined with other techniques for myocardial protection such as cold crystalloid cardioplegia or cold blood cardioplegia. Cellular swelling and edema is a nearly universal finding following cooling and warming of living tissue, and this is a predictable accompaniment of total circulatory arrest with profound hypothermia. Mild dysfunction of multiple organ systems is common. The reversal of this effect may be accelerated by vigorous postoperative diuresis. There is often a temporary rise of BUN and creatinine in the early postoperative interval after total circulatory arrest, and diuretics are often necessary to augment urine flow.

Diabetes insipidus is not a predictable side effect of total circulatory arrest. Neuronal activity is recovered at a higher temperature than the temperature at which it disappeared during the cooling process. By the first postoperative day there is generally no evidence of delayed synaptic transmission, and mental confusion usually has cleared. Myocardial depression is generally not a problem with total circulatory arrest intervals of 60 minutes.

92
Q

The duration of safe global myocardial ischemia is prolonged by infusion of cardioplegic solutions, the most important characteristic of which is?

A

Hypothermia - compared to all other elements of cardioplegic arrest, temperature is clearly the most important in allowing prolonged ischemia.

The other characteristics of cardioplegia solutions have been shown to enhance the tolerance of the myocardium to ischemic results. The essential principles are to achieve a rapid diastolic arrest as soon as possible after the induction of ischemia using a high (25-30 mEq/L) potassium concentration. During the ischemic period, only anaerobic metabolism occurs. Therefore; all solutions contain a metabolic substrate such as glucose. The ischemic or injured myocardium becomes especially susceptible to calcium overload that may enhance cellular injury and result in postischemic muscle dysfunction. Several techniques to provide an oxygen source during cardioplegia (e.g., blood cardioplegia or oxygenated crystalloid cardioplegia) have been shown to enhance the tolerable duration of ischemia.

93
Q

During deep hypothermic perfusion, what is referred to as alpha stat pH regulation (as opposed to pH stat regulation)?
How is it managed?

A

The optimal pH for patients undergoing cardiac surgery during deep hypothermia has not been determined. The accepted normal value for humans, i.e., a pH of 7.40 ± 0.05, is measured at a blood temperature of 37C and may not be appropriate as the body temperature is lowered. The regulation of arterial pH during these procedures is accomplished by one of two commonly used methods. One method considers a pH of 7.40 to be optimal at ALL temperatures. This is “pH stat” regulation.

According to Rosenthal’s equation where there is an increase in pH of 0.015 for each degree centigrade of hypothermia, a pH of 7.40 is optimal ONLY at 37C. This is termed “alpha stat” regulation. In an alpha stat or regulatory mode, a pH of 7.40 at 20C would represent a significant acidosis when corrected to 37C, i.e., the pH would be 7.145. Groups that practice pH stat regulation either decrease minute ventilation or add CO2 to the gas mixture in the oxygenator to achieve the desired degree of respiratory acidosis. When using alpha stat regulation, minute ventilation is maintained at normal normothermic values to produce a relative alkalosis as the temperature is lowered. Regulation of pH is necessary regardless of whether initial surface cooling is used prior to perfusion or whether all cooling and rewarming is done by high flow cardiopulmonary bypass.

94
Q

After total circ arrest, rewarming preliminary to the termination of cardiopulmonary bypass may best be facilitated by?

A

Rewarming may be facilitated by pulsatile flow - easily accomplished after the heart has resumed a reasonable rhythm, by transfer of volume from the bypass circuit to the heart to allow cardiac ejection. This pulsatile flow opens capillaries in the periphery and allows effective transfer of heat from the blood compartment of the body to the tissue compartment.

The use of phenylephrine to increase perfusion pressure is counterproductive. This drug will vasoconstrict the patient, thereby decreasing peripheral tissue perfusion and decreasing the effectiveness of heat transfer. Heated humidified oxygen will not be helpful with heat transfer because the flow of blood through the lungs on cardiopulmonary bypass is limited. This technique might be beneficial if the patient remains cold after cardiopulmonary bypass is discontinued. Warming of the blood in the bypass circuit to 37C during the period of circulatory arrest results in only a limited transfer of heat and will have only a temporary effect on total patient warming.

95
Q

Blood-based solutions, particularly with reperfusion modifications, have been shown to have a clear advantage over crystalloid cardioplegia solutions in situations in which the myocardium is compromised by ischemia prior to the onset of the crossclamp interval.
What properties make this true?

A
  • Provides antioxidants and free-radical scavengers.
  • Not complex to prepare.
  • Provide physiologic PH and electrolytes.
  • Limits hemodilution when large volumes of cardioplegic solution.
  • Provides excellent buffering and osmotic properties.
96
Q

A 69-year-old man with an ejection fraction of 30% is undergoing a 2-vessel CABG for a proximal 90% lesion in the left anterior descending artery and multiple 85% lesions in the right coronary artery. Monitoring was initiated with a peripheral arterial and pulmonary artery catheters. A two-stage cannula was placed in the right atrium and direct arterial cannulation is accomplished in the ascending aorta. Immediately upon placing the antegrade cardioplegia catheter, the aorta distal to that point turns blue and dilates. The mean arterial blood pressure falls to 30mmHg (right radial artery catheter). The optimal immediate response includes?

A

Leaving the aortic cannula in place and exposure of the femoral artery for cannulation.

An acute type A aortic dissection has occurred due to intimal or medial disruption from the cardioplegia catheter. Immediate repair of the injury is warranted. Aortic stent-grafting techniques, while potentially an option for descending type B aortic dissections, are neither appropriate nor available for treatment of a type A dissection in this setting. Initiation of cardiopulmonary bypass via the current ascending aortic cannula is unwise since its position cannot be assured in the true lumen of the aorta without initiating flow. The fall in radial artery pressure signifies involvement of the innominate artery. This precludes axillary cannulation in most cases. The femoral artery is the most appropriate location for arterial inflow while cooling for ascending aortic replacement. Femoral inflow should be established before removing the arterial cannula, since its removal at systemic pressures may lead to uncontrollable bleeding. The described situation is a true emergency, and hopefully, the dissection can be repaired by ascending aortic replacement alone. Femoral cannulation to establish bypass allows isolation of the heart and ascending aorta for cardioplegia and aortic repair. Chances are good that retrograde perfusion from the groin catheter will favor the true aortic lumen. Profound hypothermia and circulatory arrest should be considered for arch assessment, for restoring the integrity of the wall and for performance of the distal ascending aortic graft anastomosis.

97
Q

A 65 year old man is to undergo aortic valve replacement for critical aortic stenosis. Preoperatively, he has concentric left ventricular hypertrophy by echocardiogram, echocardiogram and chest x-ray. Operative technique is to include venous drainage by a two-stage right atrial cannula, aortic perfusion and left ventricular venting via the superior pulmonary vein. Myocardial protection technique is intermittent antegrade cold blood cardioplegia, topical hypothermia and systemic hypothermia.
The region of the heart most likely to be inadequately cooled is what?
Why?
What are the consequences?

A

Atrial septum.

Protection by hypothermia is the principle here. The entry of relatively warm blood into the cardiac chambers is the major cause of cardiac rewarming during cold, cardioplegic arrest. The atrial septum is the region most susceptible to warming because it is inaccessible to topical irrigation, particularly if venous return enters the heart.

Postoperative tachyarrhythmias and conduction disturbances have been attributed to inadequate protection of the atrial septum, as this is the location of the specialized conduction system.

Rewarming of the left ventricular subendocardium by bronchial and collateral flow can be limited by lowering the systemic perfusate temperature and venting. Although the right ventricular free wall is often exposed to ambient air and the operating room lights, effective venous drainage and topical hypothermia can prevent significant warming.

98
Q

The most likely etiology for a focal neurological deficit following coronary revascularization?

A

Atheroembolism.

99
Q

A 57-year-old man underwent emergent repair of an anteroapical ventricular septal rupture by an infarct exclusion technique. Neither coronary angiography nor coronary bypasses were performed. He asks what to expect in the future.

A

After five years between 66-88% of hospital survivors are alive and well. Most are NYHA class I or II. Additional procedures are not routinely required.
Survival is dependent on preop CAD, postop renal failure, and residual shunt.
Concurrent CABG has not proven to add longevity.

100
Q

A patient with symptomatic coronary artery disease needs bypass surgery. One year ago he suffered a right hemispheric stroke, but has completely recovered. As part of the catheterization, the cardiologist also performed a cerebral arteriogram and found a 90% right ICA stenosis.
Is this considered symptomatic carotid disease?

A

Yes. He has symptomatic carotid and coronary disease.
He needs a combined procedure. For the combined procedure, the carotid should be done first to reduce the stroke risk from the CPB run.

101
Q

Is thromboemolism common in ventricular aneurysm?
Association with ventricular arrhythmias?

A

No, despite mural thrombus being presenti in 50%.

Other characteristics: ventricular arrhythmias in 15-30%, endocardial surface is smooth and non-trabeculated. Further dilation is uncommon after 6 months.

102
Q

Randomized trials of coronary angioplasty vs coronary artery bypass surgery for the treatment of selected patients with multi-vessel coronary artery disease have shown with a one to three year follow-up that?

A

The patients in the angioplasty group have undergone significantly more re-interventional procedures, either repeat angioplasty or coronary bypass surgery.

There are at least six randomized prospective trials comparing the outcome of patients with multi-vessel coronary disease treated with angioplasty or bypass surgery. There is some variability in the inclusion criteria but in all studies less than 10% of patients who were screened were randomized. In general, these studies have included patients with relatively good angiographic risk characteristics. None of the patients had left main disease and a minority of patients had triple vessel disease. The longest follow-up in any of these studies was less than three years. The results are fairly consistent. There have been no significant differences in in-hospital or late mortality.

103
Q

Indications to treat ventricular aneurysm.

A

Recurrent VT, Anticoag resistant thromboembolism, false aneurysm formation, CHF, early rupture.

A ventricular aneurysm occurs as a complication in 10%-15% of patients following myocardial infarction. Transmural infarction with extensive tissue loss develops into a fibrotic scar that dilates over time. Increased left ventricular diameter results in increased wall tension and a lowered threshold for subendocardial ischemia and angina. The large blood volume in this dyskinetic segment decreases effective stroke volume and can lead to chronic congestive heart failure. Ventricular arrhythmias are commonly associated with ventricular aneurysms. Mapping may localize the arrhythmia to the scar and its surrounding myocardium which may have re-entrant pathways or automatic ectopic foci. Procedures which ablate these pathways combined with resection of aneurysm scar have been moderately effective in relief of these arrhythmias. It is very unusual for a true ventricular aneurysm to rupture after scarring has occurred (late). However, in the acute phase of the infarction, rupture may occur and be contained by the pericardial sac. Diagnosis of such a false aneurysm mandates immediate treatment.

104
Q

Should concomitant CABG be done in patients with posteromedial papillary muscle rupture from CAD?

A

Catheterization should be done preoperatively if the patient is stable enough, and concomitant CABG is associated with better outcomes.

The generally successful operative strategy is mitral valve replacement with a low-profile mechanical valve plus indicated revascularization.

While perioperative mortality is high (19-31%), good long-term outcomes are reported for operative survivors, with most patients in NYHA Class I or II. The reoperation rate is low, mainly for valve-related complications. Preoperative factors associated with early mortality include renal failure, requirement for inotropic support and preoperative mechanical ventilation.

105
Q

A 52-year-old man had a history of chronic stable angina, hypertension, hypercholesterolemia, Type II diabetes, and cigarette smoking. Angina increased, and following a positive exercise stress test a cardiac catheterization was performed. Lesions were identified in a proximal circumflex marginal branch (80%) and the mid-right coronary artery (60%). Ejection fraction was 55%. The most appropriate therapy is?

A

The patient described has no left main or left anterior descending coronary lesions. Two-vessel disease and preserved left ventricular function were demonstrated. For this low-risk patient, most randomized trials have shown no significant benefit from revascularization, whether by surgery or catheter-based intervention. Reduction in reversible risk factors such as hypertension and diabetes, together with an aggressive cholesterol lowering regimen, cessation of cigarette smoking, and pharmacologic management of myocardial ischemia are the most appropriate therapies for this patient. Transmyocardial laser revascularization has no role in this patient’s situation and should be applied only when there are no other revascularization options. Repeat catheterization or stress testing after six months without specifying an active treatment plan is inappropriate.

106
Q

Compared to patients operated upon one to four weeks following an acute myocardial infarction, patients who require coronary artery bypass grafting within the first seven days have a significantly increased risk for what?

A

Following an acute myocardial infarction (AMI), particularly a transmural anterior infarction, a patient who requires coronary artery bypass grafting (CABG) on an urgent or emergency basis within seven days of the AMI due to persistent, medically refractory unstable angina has a high operative mortality risk (8%-10%). The operative mortality for patients undergoing coronary revascularization 8-30 days following an AMI is approximately 3%. In addition to an increased mortality risk, patients requiring CABG within seven days of an AMI, have significantly increased morbidity risks for postoperative renal failure, respiratory insufficiency, and low cardiac output requiring the use of the intra-aortic balloon pump for hemodynamic support. The incidence of neurological complications and reoperations for bleeding postoperatively are higher among patients having CABG within seven days of an AMI than under elective circumstances. However, the increased risk of these complications appears to be maintained up to 30 days following AMI.

107
Q

Off-pump coronary artery bypass grafting was recommended and accepted by the patient. Intraoperatively, the LAD is found to be intramyocardial, although visible at the apex where its diameter is 1mm. An epicardial 1.5mm diagonal branch is identified. Adequate stabilization is achieved and the epicardium is dissected with the heart beating, but the proximal LAD cannot be found. The mammary pedicle is long enough to reach the apex. The best intraoperative decision is?

A

Conversion to cardiopulmonary bypass to graft the mammary to the proximal LAD; vein grafts to the other targets.

The intent to treat a patient with an off-pump approach should never compromise the quality of the patient’s operation. Anastomosis of the mammary to the very distal LAD in the absence of mid LAD disease is suboptimal, since the smaller distal LAD introduces greater technical demands. In addition, the distal portion of the mammary has been shown to be more muscular than its mid portion, smaller in caliber and more likely to go into spasm. If the ideal segment of the LAD cannot be identified with the heart beating, cardiopulmonary bypass with cardioplegic arrest should be employed, thus facilitating identification of the intramyocardial portion of the LAD. Other anatomic factors that weigh against off-pump CABG include target vessel diameter under 1.25mm, calcification, poor quality, and multiple stenoses. The most frequent reasons for conversion to on-pump CABG are hemodynamic instability, a small target vessel, and inadequate visualization. Several studies indicate that relative inexperience with the off-pump technique is the major determinant of the conversion rate, although a patient history of congestive heart failure may also be significant. These papers indicate that patients converted from off-pump to on-pump had a higher morbidity rate (including stroke). The mortality rate in the converted group was up to six times higher than matched patients who had their procedure initiated using conventional cardiopulmonary bypass support. Urgently converted patients had a higher incidence of postoperative cardiac arrest, multisystem organ failure, vascular complications, and perioperative myocardial infarction. Experience and good judgment will minimize undesirable outcomes.

108
Q

According to multiple database studies, what risk factor within the studies was associated with the highest risk of postoperative mortality after CABG?

A

preoperative shock (requiring drugs or intra-aortic balloon counterpulsation to support blood pressure)

renal failure, recent MI, dec ventrifular function, and reoperation are also cited

109
Q

Is acute ischemic MR always an emergency?

A

No.

For the papillary muscle ruptures w/ severe MR, emergency surgery is indicated.

But mild to moderate MR can be managed by medical therapy.

110
Q

A 65-year-old man underwent angioplasty and stent placement in the proximal left anterior descending (LAD) coronary artery. Cath also showed tight stenosis of the mid circumflex coronary artery not amenable to percutaneous coronary intervention and an occluded, dominant right coronary without a graftable target vessel. He was given a bolus of abciximab during the procedure followed by a continuous infusion. Unrelenting chest pain with ST segment elevation developed two hours later and repeat coronary angiography demonstrated an 80% stenosis at the distal aspect of the LAD stent. Intravenous nitroglycerin was begun, an intra-aortic balloon pump (IABP) was placed, and cardiac surgery consultation was requested. Currently, examination reveals a pain-free patient with a systolic blood pressure of 95 mmHg and a heart rate of 90. There is no leg ischemia distal to the IABP. ECG shows anterior wall ischemia.
What do you do to manage?

A

DC abciximab and recommend conventional CABG immediately.

Abciximab binds to the IIb/IIIa platelet receptor and prevents platelet aggregation. Usually a bolus is given and is followed by a continuous infusion for 12 hours. After the infusion is stopped, acceptable platelet function usually returns within 48 hours even though the bleeding time often remains prolonged. Measurement of the bleeding time appears to be of little value in this setting and certainly should play no role in the timing of surgery. In the EPILOG and EPISTENT trails, patients receiving abciximab did not have excessive bleeding after emergency CABG, although platelet transfusions were usually necessary. Several other IIb/IIIa receptor antagonists have shorter durations of action and have not been as carefully studied in trials. Clearly this patient should have prompt surgical revascularization. Avoiding cardiopulmonary bypass (CPB) is attractive because bleeding problems might be minimized. Without CPB, however, exposure of the circumflex system in this patient with ongoing ischemia and limited target vessels may prove hazardous. A conventional CABG will allow a relatively safe and complete revascularization in this setting.

111
Q

A 56-year-old man was referred for surgical evaluation. He has a 2-year history of intermittent chest pain and dyspnea, with worsening of these symptoms over the last month. Cardiac catheterization reveals three vessel disease, with significant stenoses of the left anterior descending, right coronary, and left circumflex arteries. Echocardiography reveals an ejection fraction of 40% and moderate mitral regurgitation with no obvious mitral leaflet pathology.

What do you do?

A

Moderate mitral regurgitation in the setting of global myocardial ischemia, reduced ejection fraction, and normal valve morphology will likely improve with coronary revascularization, either alone or in combination with mitral ring annuloplasty. Although some have questioned the relative longevity of mitral repair versus replacement in these patients, there is no evidence that mitral replacement is superior to repair in ischemic mitral regurgitation (particularly in this middle-age patient).

In patients with coronary artery disease, the degree of mitral regurgitation is predictive of survival, and in patients undergoing CABG, long-term survival is inversely proportional to the degree of postoperative mitral regurgitation. Several studies have shown excellent long-term survival, in the range of 70 to 80% at five years, in patients with preoperative left ventricular dysfunction and MR who underwent either CABG alone or in combination with a mitral repair. When performing a combined CABG and mitral procedure, the distal anastomoses should be constructed before the mitral operation, since elevation and positioning of the heart may be more difficult and risky with a mitral prosthesis in place. This risk may be greater if a “rigid” annuloplasty ring is used.

112
Q

A 70-year-old insulin-dependent diabetic man presented with unstable angina 10 years after 2-vessel CABG. Saphenous vein grafts were initially placed to the right coronary and left anterior descending arteries. Cardiac catheterization reveals 75% stenosis in the LAD graft, but the RCA graft appears normal. The patient also has a history of severe phlebitis in the remaining leg vein. The best operative strategy?

A

Graft the left internal mammary artery to the LAD and leave the existing LAD and RCA vein grafts intact.

Controversy exists as to whether or not old vein grafts that are angiographically disease-free should be replaced at late reoperation. Proponents of routine replacement note that atherosclerosis is common in grafts more than five years old, even if they look normal angiographically. Others have shown that a conservative approach (replacing only diseased vein grafts) is associated with low operative mortality, good long-term survival, and a low incidence of late clinical ischemia. Regardless, routine vein graft replacement should only be considered if an appropriate conduit is available. Cryopreserved human saphenous veins have extremely poor long-term patency rates. If a marginally stenotic vein graft is ligated and replaced with only an internal thoracic artery, severe hypoperfusion with hemodynamic compromise can occur. Two options exist after grafting the internal thoracic artery to the LAD: leave the stenotic vein graft intact, or ligate the stenotic vein graft and place a new vein graft in addition to the arterial conduit, to the same coronary artery. Staged use of the internal thoracic arteries, one at the first and another at the second operation, may be acceptable in selected patients with diabetes mellitus. Skeletonization of the arteries may also decrease the risk of deep sternal infections. However, in most series, simultaneous bilateral internal thoracic artery grafting in diabetic patients significantly increases the risk of sternal wound complications and should be avoided.

113
Q

REOPERATIVE CABG offers better outcomes than PCI for which patient populations?

A

late vein graft stenosis (occurring later than 5 years after the initial operation), multiple stenotic grafts, diffuse atherosclerotic vein disease and stenotic vein grafts to the left anterior descending artery (LAD)

114
Q

What is the difference in mortality b/w anterior and posterior postinfarct ventricular septal wall rupture?

A

Anterior ruptures result in approximately 15% hospital mortality, whereas posterior defects result in a hospital mortality approximately twice this large.

115
Q

In repairing a posterior postinfarct ventricular septal wall rupture, how is it different vs an anterior defect in regards to the LV free wall.

A

The posterior defect differs from the anterior defect in that a patch repair of the left ventricular free wall is often necessary to avoid undue tension upon the ventricular suture line.

Because the right coronary artery supplies the right ventricular as well as the left ventricular free wall, a posterior infarct often involves both right and left ventricular free walls.

116
Q

How does a posterior postinfarct ventricular septal wall rupture differ vs an anterior defect in regards to atrioventricular node conduction problems?

A

Because postinfarction defects often involve the right coronary artery, they are more frequently associated with atrioventricular node conduction problems than anterior postinfarction ventricular septal ruptures.

117
Q

What is a known sensory nerve complication of radial artery harvesting? What nerve is in danger, what is the effect of injury, and how do you avoid it?

A

Sensory nerve paresthesia or anesthesia occurs in approximately 3-12% of patients who undergo radial artery harvesting.

The lateral antebrachial cutaneous nerve supplies the skin of the radial side of the forearm. Typically, the incision is made over the artery with a slight medial curve, which allows the antebrachial cutaneous nerve and the brachioradialis muscle to be reflected laterally.

The superficial radial nerve travels lateral and deep to the proximal and middle thirds of the radial artery. The superficial radial nerve is sensory for the dorsum of the hand and a small portion of the lateral thenar eminence and is the nerve that is most often injured during radial artery harvesting.

Neither of these nerves supports motor function; therefore, muscular atrophy does not develop if they are injured.

118
Q

Post MI, is ventricular free wall more common or ventricular septal rupture?

A

Ventricular free wall rupture is 10 times more common than ventricular septal rupture and accounts for almost two-thirds of early deaths after infarction.

119
Q

You are called to the cardiac catheterization laboratory to evaluate a 72 year old man in the midst of an angioplasty. Prior to angioplasty, the patient had a total occlusion of the proximal circumflex coronary artery and a 70% stenosis of the right coronary artery (RCA). While the cardiologist was attempting to open the circumflex coronary artery, the patient developed pulmonary edema and hypotension. Blood pressure is now 90/60 mmHg on low-dose dopamine and the pulse oximeter reads 93% (40% oxygen mask). A possible dissection at the origin of the circumflex coronary artery is visible. Your recommendation should be?

A

emergency operation with rapid institution of cardiopulmonary bypass and vein grafts to the circumflex and right systems and a mammary graft to the left anterior descending coronary artery

Angioplasty misadventures are some of the more common emergency situations faced by cardiac surgeons. This patient presents a probable dissection at the origin of the circumflex coronary artery, raising the possibility that the left main coronary artery is compromised. This is a potentially fatal complication. Percutaneous techniques should be abandoned. A stent should not be used in the proximal circumflex coronary artery this close to the left main coronary artery. Rapid operation is appropriate. In this situation bypass grafts should be performed to the left anterior descending coronary artery as well as the circumflex because of the potential compromise of left main coronary artery. The right coronary artery is stenotic and also should be grafted. Some might recommend that intraaortic balloon counterpulsation be initiated in the cath lab prior to transfer, recognizing that this may be of limited effectiveness with left main dissection. Although preoperative pulmonary edema increases the risk of excessive interstitial pulmonary fluid and difficulty with oxygenation in the postoperative period, the circumstances which are described will not allow placement of an intraaortic balloon pump and waiting for three days to allow resolution of pulmonary edema.

120
Q

A 62 year old woman had an anterior infarction eight days previously. At the end of an uneventful hospital course, an echocardiogram was performed. A 15 mm pedunculated mass, attached to the anterior wall near the apex, was seen within the left ventricle.
What happened?
The next appropriate step should be?

A

A 15 mm pedunculated mass within the left ventricle one week after an anterior infarction is almost certainly a thrombus related to the infarct.

The appropriate course of therapy is anticoagulation and repeat echocardiography. Over a course of four to eight weeks, these postinfarction left ventricular thrombi usually melt away without clinical evidence of systemic emboli.

Mural thrombi are not themselves an indication for operation, even when they are pedunculated.

The indication for operation upon a mural thrombus is recurrent systemic emboli despite adequate anticoagulation therapy.

An attempt at thrombolysis would be hazardous because the thrombus might partially lyse with resultant extensive systemic emboli.

121
Q

An 86 year old dialysis dependent diabetic male develops chest pain which is promptly relieved after administration of intravenous nitroglycerin and heparin. An angiogram is performed, which shows a 99% obstruction of the Left Main coronary artery, no other obstructive coronary artery disease and a porcelain calcification pattern on the ascending aorta and aortic arch. The best operative approach is?

A

Off pump coronary artery bypass with LIMA to LAD.

The major risk in this revascularization attempt is the atheromatous aorta. Any attempt to manipulate the aorta (full clamp, side biting clamp or aortic connector) increases the risk of distal embolization or dissection. Consequently, an SVG to the LAD originating from the ascending aorta is not safe. LIMA to LAD either off pump or on pump with peripheral arterial cannulation (axillary or femoral) and no cross clamping are the best options in this case. The patient’s advanced age, severe left main disease and atheromatous aorta are significant risk factors but certainly not contraindications to emergent coronary artery bypass grafting.

122
Q

A 70 year old man was admitted to the Coronary Care Unit with a diagnosis of an extensive acute anterior myocardial infarction. Angiography revealed a left anterior descending coronary occlusion, 80% stenosis of the circumflex, and anterior wall akinesis. On hospital day 5 the patient experiences sudden hemodynamic deterioration: blood pressure 80/50 mmHg, heart rate 110 beats/minute (sinus). The single best diagnostic study at this time is?

A

Echo.

Coronary care units have lessened the mortality from acute myocardial infarctions. Rapid diagnosis of the potential complications of infarction have allowed earlier treatment to be instituted. These complications include arrhythmias, infarct extension or expansion, and myocardial rupture (ventricular septum, free wall, or papillary muscle). Bedside two-dimensional echocardiography with Doppler techniques is widely available and provides the most rapid means of differentiating the life-threatening mechanical complications. It also provides a semi-quantitative analysis of ventricular function. Although the other procedures listed may also provide some important information, they do not eliminate all potential diagnoses and, therefore, are limited as rapid screening procedures.

123
Q

A 65-year-old man with normal left ventricular function underwent an uneventful coronary artery bypass and transmyocardial laser revascularization procedure. Which of the following is the most likely postoperative complication?

Bleed
Block
Low CO
Ventricular ectopy
MV injury

A

Ventricualr ectopy.

Transmyocardial laser revascularization (TMLR, TMR) may provide relief of angina for severely symptomatic patients with coronary disease not amenable to conventional therapies. In this procedure several dozen transmyocardial laser channels are created through the left ventricular wall. The exact mechanism that produces angina relief is not known, but angiogenesis most likely plays a role. Most patients selected for TMR should have reasonably well-preserved ventricular function since a degree of myocardial compromise results from creation of the transmural channels. Following hybrid CABG/TMLR most patients do well. Operative mortality is less than 5% and the most frequent postoperative complication is ventricular ectopy. 10-15% of patients will require drug treatment for ventricular ectopy. Because of this, many surgeons begin prophylactic lidocaine infusion during the procedure and maintain patients on this medication through the first postoperative day. Atrial fibrillation is infrequently seen and heart block is not directly related to either CABG or TMLR. Significant bleeding is distinctly uncommon and patients seldom require transfusion. Damage to the mitral valve has been reported, but it is quite rare. Since selected patients have good ventricular function, postoperative pump failure or low output syndrome requiring inotropic support is unlikely.

124
Q

A 70-year-old, 70 kg man had a left internal mammary graft to his left anterior descending coronary, a vein graft to his right coronary, and a sequential radial graft to two obtuse marginal coronaries. One hour after arrival in the ICU his cardiac index dropped below 1.6 L/(min*m2) despite adequate filling pressures. Mean arterial pressure was 80 mmHg and pulmonary artery diastolic pressure was 20. The EKG revealed new ST segment elevation (3 mm) in the lateral precordial leads. The most appropriate initial management is?

A

nitroglycerin drip at 0.5 mcg/(kg*min)

Although prone to vasospasm, the radial artery (RA) is commonly used as a graft in coronary artery bypass surgery. Achieving long-term patency of a RA graft requires special attention to manage vasospasm during and after operation. Atraumatic harvesting and handling, intraoperative dilatation with topical vasodilators and intravenous vasodilators perioperatively are all useful techniques to prevent vasospasm. Selection of target vessels with significant proximal stenoses and good run-off will also help prevent postoperative spasm. Inotropic agents may induce vasospasm and should be avoided if possible. Recent experimental and clinical studies have demonstrated nitroglycerine to be a superior vasodilator to diltiazem. If systemic blood pressure is adequate the starting dosage for IV nitroglycerine is 0.5-2 mcg/(kg*min). The combination of calcium antagonist and nitroglycerine appears to be more effective than either alone. If EKG changes persist despite maximal medical therapy, cardiac catheterization with intraluminal injection of vasodilators should be considered.

125
Q

Gross heart failure and new onset mitral regurgitation immediately following myocardial revascularization (after coming off CPB) on TEE is secondary to what?

A

implies inadequate blood flow to the myocardium

the initial response of the operating surgeon should be to go back on bypass and re-evaluate (and most likely to re-graft) the coronary arterial system supplying the dysfunctional myocardium indicated by TEE

126
Q

In a 55 y/o male patient who requires reoperative CABG, the best management of a diffusely atherosclerotic vein graft to the LAD is?

A

In-situ LIMA to LAD with preservation of old vein graft.

Although there is a risk of distal embolization if an old vein graft is left in place, in this situation the best option is to leave the old vein graft in place and add an in-situ LIMA to the LAD distally. Removal of the vein graft may result in hypo-perfusion if the arterial graft cannot generate a flow comparable to that of the vein graft. The risk of competitive flow and failure of the LIMA graft is low when the degree of stenosis in the vein graft is severe. A new vein graft to replace an old one may therefore result in serious competitive flow. Using a new vein graft instead of a LIMA, although is possible and used by many, does not provide the longevity advantage of a LIMA to LAD graft. There is limited evidence on other non-LITA conduits to support their routine use in this situation.

127
Q

A 74 year old woman suffered an anterior infarction with congestive heart failure four days prior to transfer from another hospital. Twenty-four hours prior to transfer, she developed a new systolic murmur and cardiogenic shock. Upon arrival she was found to be neurologically unresponsive and had been anuric for twelve hours.

The development of a new systolic murmur and cardiogenic shock four days after a myocardial infarction suggests the development of what two possible pathologies?

How do you differentiate?

What should you do for her?

A

postinfarction ventricular rupture defect or acute mitral regurgitation

Swan-Ganz catheter with evaluation of a possible stepup in oxygen saturation between the right atrium and pulmonary artery allows the clinician to differentiate between these two events

intra-aortic balloon pump placement, resuscitation, and diagnostic evaluation in the intensive care unit with reevaluation in two to three hours.

If the patient is found to have a postinfarction ventricular septal rupture, hemodynamic stabilization with pharmacologic support, and an intra-aortic balloon pump is the most appropriate course, followed by cardiac catheterization and urgent operation. One should determine the coronary pathology by coronary angiography prior to proceeding to the operating room, but a ventriculogram is not necessary and the extra dye load may be harmful. If the patient is found to have acute mitral regurgitation, pharmacologic management and intra-aortic balloon pump placement is ordinarily the initial therapy. Some patients may improve without urgent operation, but most patients in cardiogenic shock with papillary muscle dysfunction and all patients in cardiogenic shock with rupture of the papillary muscle are best managed by urgent mitral valve repair or replacement and coronary revascularization. Once again, coronary angiography should be accomplished prior to operation.

This 74 year old woman who is neurologically unresponsive and who has been anuric for 12 hours may be among the group of patients with multisystem organ impairment for whom emergency operation for cardiogenic shock is not indicated. Particularly, the presence of anuria for 12 hours suggests that the salvage rate from emergency operation will be extremely low. If the patient should improve after resuscitation and diagnostic evaluation in the intensive care unit, then she may be a candidate for operation. This patient should be resuscitated and reevaluated before rushing to the operating room.

128
Q

Augmented glucose uptake by positron emission tomography has been demonstrated to be associated with what in regards to recoverable function of myocardial ischemia?

A

Augmented glucose uptake by positron emission tomography has been demonstrated to be associated with functional recovery following revascularization of previously akinetic segments.

Augmented uptake of technetium pyrophosphate is indicative of myocardial infarction. Reversible defects on thallium 201 scans are interpreted as areas of viable myocardium and have been shown to have potential for function recovery. Experimental studies suggest that lipid detection in the area of ischemic myocardium by magnetic resonance imaging is indicative of viability. Although not absolutely specific, ST segment elevation during an anginal attack is classic evidence of ischemia and strongly suggests the potential for surgical correction.

129
Q

What about augmented uptake of technetium pyrophosphate on radionuclide scan means what in regards to diagnostic eval of myocardial ischemia?

A

Augmented uptake of technetium pyrophosphate is indicative of myocardial infarction.

130
Q

In terms of myocardial viability, what are thallium scans used to show?

How can lipid detection be used in cardiac MRI for myocardial viability?

ST segment elevation during an anginal attack is suggests what in terms of correction?

A

Reversible defects on thallium 201 scans are interpreted as areas of viable myocardium and have been shown to have potential for function recovery.

Experimental studies suggest that lipid detection in the area of ischemic myocardium by magnetic resonance imaging is indicative of viability.

Although not absolutely specific, ST segment elevation during an anginal attack is classic evidence of ischemia and strongly suggests the potential for surgical correction.

131
Q

Autologous blood donation reduces the need for homologous blood transfusions in cardiac surgery. Blood can be collected safely from most patients in whom a temporary decrement in intravascular volume can be tolerated.
Which types of cardiac lesions is predonation most suited for/not suited for?

A

Predonation is feasible in volume loading lesions such as aortic regurgitation, left ventricular aneurysms, and mitral valve disease. In a patient with aortic stenosis a sudden decrement in blood volume could lead to a sudden decrease in cardiac output. Autologous blood donation should be avoided for a patient with this lesion.

132
Q

Transfer of acetylated fatty acids from the cytosol to the mitochondria for subsequent beta-oxidation requires the presence of adequate levels of?

A

Carnitine, a water-soluble, naturally occurring amino acid, is the requisite carrier for transport of these fatty acids from the cytosol across the mitochondrial membrane for beta oxidation.

Ischemic depletion of carnitine results in the toxic accumulation of esterified fatty acid intermediates. These metabolites inhibit a variety of critical intracellular enzymes involved with regulation of cell volume, maintenance of ionic gradients, and nucleotide transfer. Fatty acid metabolites are also deleterious because they activate phospholipases and have intrinsic detergent properties that nonselectively alter subcellular membranes.

Long chain fatty acids are the preferred metabolic substrate for oxidative metabolism in the heart. After passive transfer from the plasma to the cytosol, fatty acids must first be activated to fatty acetyl-coenzyme A subsequent to further metabolism. Although some activated fatty acid is esterified to triglyceride, the majority serves as substrate for oxidative phosphorylation.

133
Q

A 65 year old man with normal preoperative left ventricular function has just returned to the intensive care unit following coronary artery bypass grafting. His intraoperative course was unremarkable. His initial hemodynamic and laboratory data includes: Pulse: 75 beats per minute Pulmonary artery pressure: 35/17 torr Hemoglobin: 11 gm/dL Pulmonary artery wedge pressure: 13 torr Blood pressure: 130/84 torr Cardiac index: 1.7 L/min^m2 Systemic vascular resistance index: 4100 dynes sec cm -5/m2 The most appropriate initial treatment of this patient is?

A

This patient has a high systemic vascular resistance, hypertension, and a low cardiac index. The appropriate initial management for this patient is to lower his afterload. This will improve his cardiac index and lower his myocardial oxygen consumption by decreasing wall stress. Sodium nitroprusside is an effective arterial and venous vasodilator that is commonly used to control hypertension and lower afterload in the postoperative setting. Volume, initially, is not indicated in this patient because his pulmonary capillary wedge pressure is adequate. His principal problem is not hypovolemia, but increased afterload. The increased SVR may be in part secondary to residual hypothermia, so active rewarming should continue if appropriate. Adequate pain control must also be assured. Inotropic agents (dobutamine and epinephrine) are not indicated since they do not directly address the problem of high afterload and they increase myocardial oxygen demand. Nitroglycerin is a peripheral venodilator, and it is not as effective at reducing systemic arterial resistance as nitroprusside.

134
Q

In a mechanically ventilated patient with an arterial oxygen saturation of 95%, appropriate treatment of decreasing mixed venous oxygen saturation should focus on what?

A

Treatment of decreasing mixed venous oxygen saturation must focus on increasing venous return and/or decreasing airway pressure. Crystalloid, albumin or blood transfusion may be appropriate depending on the patient’s hemoglobin level. Decreasing the tidal volume and respiratory rate will decrease airway pressure. The inspiratory/expiratory ratio and other mechanical ventilation parameters may also be changed in an effort to decrease the mean airway pressure. However, in the absence of hypoxia, increasing the inspired oxygen concentration will not significantly increase hemoglobin saturation or oxygen delivery. The mixed venous oxygen saturation will not be effected, and high FiO2 can contribute to oxygen toxicity.

135
Q

An 80-year-old woman presents for evaluation for coronary artery bypass grafting (CABG). Her left ventricular ejection fraction is 30%. Her past medical history is significant for chronic obstructive pulmonary disease and insulin-dependent diabetes. The preoperative creatinine is 2.2 mg/dL. The most significant preoperative risk factor for acute kidney injury requiring dialysis postoperatively is?

A

Factors that increase the risk of needing postop dialysis include female gender, EF < 35%, insulin-dependent diabetes, COPD, emergency surgery, and previous cardiac surgery. Combined valve-CABG surgery carries a higher risk. However, preoperative creatinine > 2.1 mg/dL carries the highest weight in this risk prediction score.

Acute kidney injury is a known complication of cardiac surgery. The presence or development of acute kidney injury increases in-hospital mortality and also decreases overall long-term survival. The general effects of cardiopulmonary bypass and its altered hemodynamics have been implicated in kidney injury, but several specific preoperative factors in the Cleveland Clinic Risk Predictor Score predict postoperative severe kidney injury.

136
Q

A 55-year-old man presents with severe aortic stenosis. An echocardiogram shows a mean aortic transvalvular gradient of 55 mmHg and a left ventricular ejection fraction of 30%. He undergoes aortic valve replacement with a mechanical prosthesis. Intraoperatively, he receives 4 units of packed red blood cells. Ten days after surgery he is severely fatigued and anorexic. He is taking warfarin at 5 mg per day with an INR of 3.5. His hematocrit is 25%. On physical examination he has increased jugular venous pressure and pulsus paradoxus; the heart sounds are distant and regular mechanical valve opening and closing sounds are appreciated. An ECG documents sinus tachycardia and normal ST segments. His CXR shows an enlarged cardiac silhouette. The most likely diagnosis is?

A

Acute RV failure usually presents with typical findings on electrocardiogram, including indications of RV infarction or ST segment elevation. CHF may present with findings similar to those seen in this patient, but pulsus paradoxus is unusual in CHF and an abnormal third heart sound is common. Excessive blood volume expansion is ordinarily accompanied by a higher than expected hematocrit.

Delayed tamponade following a cardiac operation is related to perioperative bleeding, and the intrapericardial fluid volume increases on an osmotic basis as the blood elements break down. The described patient should have an echocardiogram, which usually demonstrates circumferential fluid in the pericardium that compresses the chambers, marked respiratory variation in transvalvular flows, and both right and left ventricular collapse.

137
Q

A 34-year-old patient with a degenerated aortic homograft and severe aortic regurgitation (AR) is undergoing reoperation. A preoperative CT frame is shown. The femoral artery and vein have been exposed. The sternum is divided, but during cautery dissection the heart fibrillates. Multiple attempts at defibrillation fail. A small portion of the heart has been dissected free. Cardiopulmonary bypass (CPB) is initiated via the femoral vessels but the left ventricle (LV) is distended. Another attempt at defibrillation is not successful. The best next step is to?

A

Left ventricular distention can lead to serious and irreversible ventricular failure after cardiac surgery. In the setting of reoperative surgery and severe AR, the ability to vent the left ventricle (LV) may be difficult. A simple and efficient solution is to vent the apex directly, particularly when dissection of the heart will be difficult and time consuming. Severe AR will prevent full decompression on CPB and can even compromise the ability to perfuse the patient. Although it may be a tempting thought to quickly dissect out the heart and aorta, the ability to crossclamp a calcified aorta (this patient’s homograft) is not realistic. Despite the presence of a LV vent and full CPB, it may be difficult to support patients with severe AR, at which point hypothermia and circulatory arrest may be the best protection strategy.

138
Q

A 64-year-old man presents with progressive dyspnea on exertion and stable angina. He has known ischemic cardiomyopathy with functional NYHA Class III congestive heart failure. His left anterior descending coronary artery is occluded and the circumflex and dominant right coronaries have high grade stenoses. The left ventricular (LV) ejection fraction is 32%. A cardiac MRI demonstrates an anteroapical LV aneurysm, a viable and functional basilar region, and viable myocardium in the inferior and lateral walls. Transesophageal echocardiography reveals mild mitral regurgitation and the LV end-diastolic volume is 60 mL. To maximize 5-year survival, the best next step is?

A

Indications to consider surgical ventricular reconstruction/restoration include anteroseptal myocardial infarction, candidacy for revascularization and mitral valve (MV) repair, good right ventricular (RV) function, symptoms of congestive heart failure (CHF), enlarged LV with a large area of akinesis/dyskinesis, and retained basilar function with overall depressed EF. Absolute contraindications for surgical ventricular restoration include viable anterior wall myocardium and active ischemia with coronary lesions that will not benefit from bypass. Diagnostic studies are chosen to define the ventricular anatomy/pathology, coronary artery disease, MV function and RV function. Thus, TEE is important for assessment of LV regional wall motion, ventricular dimensions, valvular dysfunction, and biventricular function. Coronary angiography is required to define coronary pathology, Perhaps most importantly, cardiac MRI provides 3-dimensional visualization of ventricular function and geometry as well as accurate assessment of myocardial viability (using gadolinium).

Several studies have supported the benefits of surgical ventricular restoration. Some found better long-term survival and improvements in symptoms, EF, ventricular size, stroke volume index, endocrine markers of CHF, ventricular energetics, mechanical efficiency, and the incidence of ventricular arrhythmias. The RESTORE (Reconstructive Endoventricular Surgery returning Torsion Original Radius Elliptical shape to the LV) group published the most well-known study related to surgical ventricular restoration. In this study, EF, NYHA class, and LV end-systolic volume improved postoperatively. Of note, 95% and 22% of patients underwent concomitant CABG and mitral valve repair, respectively. Preoperative risk factors that increased long-term mortality included older age, degree of CHF, EF < 35%, loss of basal function, cardiomegaly, LVED pressure > 20 mmHg, and the presence of mitral regurgitation. Residual myocardial function was a determinant of overall improvement after surgery. However, more recently the STICH (Surgical Treatment for Ischemic Heart Failure) Hypothesis 2 Trial failed to confirm such optimistic results. This part of the study compared CABG to CABG plus surgical ventricular reconstruction in a randomized controlled trial. After 48 months, the study demonstrated no difference in the primary outcomes of death and cardiac-related hospitalizations between the groups. Although the STICH study has been criticized, questions still remain about the effect of ventricular restoration on diastolic ventricular function and the actual improvement in ventricular geometry vs. global size.

139
Q

The mechanism of action of Recombinant factor VIIa (rFVIIa) is?

A

Recombinant Factor VIIa acts locally at the site of tissue and vascular wall injury by binding to exposed tissue factor. A small amount of thrombin is formed that is sufficient to activate platelets. The activated platelet surface then forms a template on which rFVIIa can directly or indirectly mediate further activation of coagulation. Eventually, more thrombin is formed and, therefore, fibrinogen is converted to fibrin.

140
Q

Experimental studies have suggested what wall of the heart is the area most vulnerable to inadequate myocardial protection during retrograde cardioplegia?

A

the anterior wall of the right ventricle

141
Q

Where does most of the oxygen consumption savings come from when doing CPB and cardioplegia?

A

Up to 80% of myocardial oxygen consumption savings is a direct result of the cessation of electrical and mechanical activity of the heart during cardioplegia, while hypothermia induces only a modest reduction in metabolic cellular activity.

142
Q

A 63 year-old man is undergoing repair of an acute Type A aortic dissection. Shortly after instituting cardiopulmonary bypass (CPB) via an arterial inflow cannula in the left femoral artery, the cerebral oximetry reading on the right drops from 75% to 20%. The left side reading remains unchanged from baseline.
What happened?
What’s the best next step and the options thereafter?

A

In the scenario presented, the etiology of the precipitous drop in unilateral cerebral oximetry is most likely due to cardiopulmonary bypass-induced malperfusion of the innominate artery from perfusion and pressurization of the false lumen by the femoral cannula. Failure to recognize this complication and proceeding with planned repair will likely result in a massive stroke. Management requires prompt recognition of the abnormalities and immediate cessation of cardiopulmonary bypass. An alternate cannulation site must be chosen to perfuse the true rather than the false lumen.

Right axillary arterial cannulation using a small diameter side graft has proven benefit for this application and has been associated with lower rates of mortality and neurological complications when utilized for repair of acute Type A aortic dissection. Management of the dissection requires rapid re-establishment of cerebral blood flow. Central aortic cannulation of the true lumen despite the dissection has been championed by several centers. Direct needle puncture, wire position confirmation by TEE, and Seldinger exchange for an arterial perfusion cannula has become routine for some surgeons. Ascending aortic transection followed by insertion of a cannula into the true lumen with a surrounding snare for hemostasis is another option that allows perfusion and cooling. A brief period of hypothermic arrest permits arch inspection for tears and re-entry points and for a distal graft anastomosis to a reconstituted aorta.

142
Q

A 62-year-old woman underwent redo CABG and postoperative progress was satisfactory. Her chest tubes were removed on POD#2 and pacing wires were removed on POD#3. On the morning of POD#5, she was found to be somnolent and markedly short of breath. Her vital signs included heart rate 110 and BP 90/60. Blood gas values were pH 7.28; pO2 85; pCO2 32; and Base Excess -5. Her urine output was 70 mL for the last shift. EKG with sinus tachycardia.
What’s next?

A

Echo

Sinus tachycardia and low systolic blood pressure, plus low urine output and acidemia on arterial blood gas analysis collectively indicate low cardiac output. A stat echocardiogram to evaluate left ventricular function and to exclude cardiac tamponade is required. This patient’s chest tubes were removed before the pacing leads were taken out, and tamponade should be suspected.

Cardiac catheterization to evaluate for myocardial infarction is not indicated in the absence of acute chest pain or ST-T wave changes on ECG. Although the patient is mildly hypoxic, the low blood pressure and low urine output suggest more is going on than isolated respiratory insufficiency. Although the patient is somnolent, there is no mention of a focal neurologic deficit and acute cerebrovascular accident (CVA) would not cause physiologic features of shock. Pulmonary embolism is quite unusual following CABG, and an embolic clot load sufficient to cause shock usually causes notable ECG changes. The second ECG graphic features such findings, including T-wave inversions in the anterior and inferior leads along with (new) right bundle branch block. This differs from the described patient’s finding of only sinus tachycardia. An echo also has diagnostic value for massive pulmonary embolism, and it is a more appropriate choice in this case before a PE protocol contrast CT is done.

143
Q

What are the most important variables to identify in patients at elevated risk of perioperative blood loss and transfusion need?

A

Advanced age (> 70 years), emergent operations, complex operations, small body size (lower red blood volume, i.e. peds), preop anemia.

Lysine analogues such as epsilon-aminocaproic acid are useful adjuncts in routine blood conservation. However, aprotinin has been associated with an increased risk of 30-day death and renal dysfunction, and it is not recommended or available for routine use. Blood loss and transfusion requirements are significantly reduced with thoracic endovascular aortic repair (TEVAR) compared to open procedures in matched groups of patients. Modified ultrafiltration has been demonstrated to reduce postoperative blood loss in both pediatric and adult cardiac operations. Drugs that inhibit the platelet P2Y12 receptor (e.g., clopidogrel) should be discontinued, whenever possible, 7 days prior to surgery in order to replete functional platelet numbers and reduce perioperative blood loss.

144
Q

A 75-year-old man underwent elective 3-vessel coronary artery bypass grafting. His preoperative left ventricular ejection fraction was 35%. He has a permanent pacemaker and moderate tricuspid regurgitation secondary to his pacemaker leads. Four hours postoperatively, his cardiac index by Swan-Ganz thermodilution is 1.3 L/min/m2. His mean arterial pressure is 65 mmHg and his urine output is 60 mL/hr. The oxygen saturations are 98% and 70% on arterial and pulmonary artery blood gases, respectively. His hemoglobin is 12 gm/dL and his body surface area is 2.0. The best next step is to

A

Calculate the Fick. His calculated Fick is normal, and nothing needs to be done about the measured low CI by thermodilution. The thermodilution calculation is inaccurate in this patient because of tricuspid regurgitation.

Classically, the oxygen consumption was determined by direct measurement of exhaled air or by metabolic cart calorimetry. Because direct measurement is cumbersome, oxygen consumption is frequently estimated by body surface area (125 x BSA). Arteriovenous oxygen difference is determined from arterial and pulmonary artery oxygen saturations and hemoglobin level: 1.36 x Hgb x (arterial O2 sat – venous O2 sat).

In the cardiothoracic ICU, CO measurement is most frequently determined by thermodilution measurements via Swan-Ganz pulmonary artery catheters. Cold saline of known temperature and volume is injected into the right atrium and a distal thermistor measures the change in temperature as blood advances through the heart and out into the lungs. An algorithm then estimates the CO based on the temperature change over time. Although this technique has the advantage of rapid measurement, it can be inaccurate in settings of tricuspid regurgitation, intracardiac shunts, irregular rhythms, low CO states, suboptimal injection techniques and incorrect catheter tip positions. Other techniques utilized to estimate CO include continuous thermal and impedance measurement. Although other techniques have been developed, the Fick principle represents the “gold standard” technique and should be utilized when thermodilution measurements appear inaccurate. Fick CO measurements will be inaccurate when all parameters are not directly measured. When oxygen consumption is estimated by BSA for the indirect Fick method (125 * BSA), an error is propagated that is most notable for obese individuals. In these cases, a metabolic cart may be required to measure the oxygen consumption more accurately.

145
Q

What are some things that can affect the thermodilution read by the Swan catheter when measuring CI?

A

tricuspid regurgitation, intracardiac shunts, irregular rhythms, low CO states, suboptimal injection techniques and incorrect catheter tip positions

In the cardiothoracic ICU, CO measurement is most frequently determined by thermodilution measurements via Swan-Ganz pulmonary artery catheters. Cold saline of known temperature and volume is injected into the right atrium and a distal thermistor measures the change in temperature as blood advances through the heart and out into the lungs. An algorithm then estimates the CO based on the temperature change over time. This is fast, but can be inaccurate if any of the above situations are true.

146
Q

How can you estimate oxygen consumption? How can you measure it directly?

A

Classically, the oxygen consumption was determined by direct measurement of exhaled air or by metabolic cart calorimetry. Because direct measurement is cumbersome, oxygen consumption is frequently estimated by body surface area (125 x BSA).

147
Q

A patient arrives in the intensive care unit still intubated immediately following aortic and mitral valve replacements surgery, with temporary atrial and ventricular pacing wires present. The mean blood pressure is 63 mmHg. The patient is on 8 μg/min of norepinephrine. Blood gases show pH 7.34, pO2 132 torr, pCO2 35 torr. The serum lactate is 7 mg/dL, and urine output for the past hour is 25 mL. The patient’s ECG is shown.
What is the diagnosis/assessment of this patient?
What is the immediate next step?

A

Left bundle branch block (LBBB) and 2:1 A-V block with a ventricular rate < 50/min.
This is causing a low index, as evidenced by his lactate level.

Use the pacing wires. DDD if there’s no good sinus atrial rhythm, otherwise AAI is ideal.

148
Q

Three days after aortic valve replacement surgery, a patient is comfortable on nasal cannula oxygen and sitting in a chair. His mean arterial pressure is 76 mmHg; he is afebrile, alert, and oriented. His urine output is 50 mL/hr. Temporary pacing wires were placed at the time of surgery due to bradycardia on initial separation from cardiopulmonary bypass. He currently is connected to the a temporary pacemaker with the following settings: mode: VVI, output: 20mA, sensitivity 6mV, rate 48/min. The bedside nurse calls a code when, without warning the patient slumps over in conjunction with the rhythm strip shown. A single rapid external shock recovers his vital signs and consciousness.
What happened?
The best next step in management is to?

A

R-on-T phenomenon.
Disconnect the temporary pacemaker. Should bradycardia become symptomatic the sensitivity setting on the temporary pacemaker should be decreased (e.g., change from 6maA to 1mA), which should enhance the device’s identification of native QRS complexes and inhibit pacing.

R-on-T phenomenon is one in which perturbations to the heart’s electrical activity during the repolarization (T wave on ECG) can produce ventricular fibrillation. This condition can occur whenever an electrical impulse stimulates the heart during the T wave. Outside the hospital setting, electrical stimulation can occur from high voltage contact. Mechanical trauma to the myocardium can elicit an electrical response and, when timed in concert with the T wave, can produce ventricular fibrillation. In the postoperative setting, temporary pacing has been associated with iatrogenic ventricular tachyarrhythmias due to the R-on-T phenomenon.

Modern, microprocessor controlled temporary pacemakers can be programmed to minimize the chances of R-on-T. In VVI mode the pacemaker monitors electrical activity inhibits delivery of pacing spikes based on a sensitivity setting. The sensitivity is a value in millivolts that will inhibit the device from firing. A high sensitivity number setting means a very large electrical signal (strong QRS signal) is needed to alert the device of intrinsic myocardial activity, and to avoid R-on-T. As sensitivity approaches infinity, the device no longer functions in VVI mode. Since it can no longer sense any physiological electrical energy from the heart, it functions as VOO.

The tracing shows 2 QRS complexes not sensed by the pacemaker, evidenced by pacing spikes (arrows) delivered on the T-wave. This paced-R-on-nativeT induced ventricular fibrillation. The patient was alert and awake when this incident occurred. The native R-R interval indicates an intrinsic rate of about 48, so the initial response after defibrillation should be to disconnect the temporary pacemaker.

149
Q

Describe glucose control in postop cardiac patients.

A

A number of studies have shown that the incidence of wound infections and other infections is decreased when blood sugar is managed with insulin in the postoperative period. The researchers found a significant decrease in deep sternal wound infections in the study arm (P = 0.005). Continuous intravenous insulin infusion reduces the incidence of deep sternal wound infection in diabetic patients after cardiac surgical procedures.

Postoperative hypoglycemia is a potential complication of postoperative blood sugar control with insulin therapy. Hypoglycemia may have an impact on postoperative mortality. Iatrogenic hypoglycemia secondary to tight glucose control is an independent determinant for mortality and cardiac morbidity. In comparisons between groups of patients treated with different target blood sugar ranges, patients treated with higher range targets have less chance of developing hypoglycemia.

The ideal target blood sugar range for postoperative cardiac surgery patients has been under debate. Early studies involving goal range of 80 to 110 that demonstrated improved outcomes were not reproduced in later studies (NICE-SUGAR). Meta-analysis and later studies also failed to show advantages of strict control. Most agree that keeping glucose <180 avoids hypoglycemia while avoiding infection.

150
Q

A 61-year-old man was given aspirin for an ST elevation myocardial infarction, and ticagrelor 180 mg was administered in anticipation of a percutaneous coronary intervention. A cardiac catheterization revealed severe 3-vessel disease with a nearly occluded left anterior descending (LAD) artery. Angioplasty attempts resulted in LAD dissection and occlusion, and emergency coronary artery bypass surgery was done. Now in the ICU the patient is hemodynamically stable on inotropic drugs, but bleeding continues at 250 mL per hour.

Describe your management strategy.

A

Coagulopathic bleeding after anti-platelet therapy prior to PCI/stent is characterized by diffuse bleeding from capillary beds at wound surfaces. Intraoperative or postoperative blood loss exceeding 250 mL/h over 4 hours or a sudden increase bleeding after the first 2 hours is frequently seen. Basic hemostatic management that should be guided by routine coagulation tests. Platelet dysfunction can be confirmed with platelet function tests, but platelet concentrates are first-line therapy. Postoperative blood loss requires RBC transfusion to maintain mean arterial pressure > 70 mmHg and hematocrit > 22-25. Fresh frozen plasma (and/or prothrombin complex concentrate) and fibrinogen (25-50 mg/kg) requirements can be determined by thromboelastography (TEG), PT, and PTT. Clinical evidence of tamponade mandates early chest exploration, as does persistent bleeding after correction of coagulation abnormalities. Recombinant factor VIIa may be considered for patients with intractable bleeding after a surgical cause has been excluded. Adherence to transfusion protocols for coagulopathy is associated with lower resource utilization and improved outcomes.

Ticagrelor is a direct acting and reversible inhibitor of the P2Y12 receptor and an inhibitor of adenosine reuptake. Like prasugrel, ticagrelor has a more rapid and consistent onset of action compared with clopidogrel. Studies suggest better outcomes in patients with acute coronary syndrome, irrespective of revascularization strategies. Ticagrelor has been recommended for all patients at moderate-to-high risk of ischemic events, regardless of initial treatment strategy. In addition, it has a more rapid and consistent end of activity related to its reversible receptor binding and plasma half-life of 6–12 hours. In patients with stable CAD, recovery of normal platelet aggregation is almost complete at 5 days after stopping ticagrelor. For patients in the PLATO study who underwent CABG within 7 days of the last dose of ticagrelor, there was reduced early mortality, fewer ischemic events (4.6% vs. 9.2%) and fewer deaths associated with bleeding and infection.

151
Q

Perioperative blood product transfusions related to cardiac surgery have been implicated as a patient safety issue and a public healthcare cost contributor. The restrictive approach to transfusion of packed red blood cells (PRBC) is?

A

Cardiac surgery is responsible for much of the blood product utilization at many institutions.

In 1999 the Canadian Transfusion Requirements in Critical Care (TRICC) trial demonstrated that a restrictive red blood cell transfusion strategy (Hgb 7.0-9.0 gm/dL trigger) was at least as effective, and possibly superior, to a liberal strategy (Hgb 10.0-12.0 gm/dL trigger), in preventing death at 30 days. These findings have been generally validated for cardiac surgery patients as well. There is wide variability in rates of transfusions of PRBCs at U.S. hospitals according to data abstracted from the Society of Thoracic Surgeons Database. Considering all cardiac procedures the probability of transfusion of PRBCs is about 60%.

Implementing blood product conservation initiatives has been shown to significantly improve outcomes, including morbidity rate, mortality rate, and resource utilization. To offer further guidance, a Task Force of the Society of Thoracic Surgeons and the Society of Cardiovascular Anesthesiologists produced Blood Conservation Clinical Practice Guidelines.

152
Q

A 59-year-old man with a body mass index of 38 underwent coronary artery bypass grafting. He has a history of insulin-dependent diabetes and chronic obstructive pulmonary disease. He is discharged to home on the fifth postoperative day and returned to emergency department two weeks later with fever to 38.9oC and chills. His heart rate is 105 beats/min and blood pressure 90/50 mmHg. The lower sternal incision is erythematous with yellow drainage, but the sternum is stable. A chest CT reveals fluid and gas posterior to the sternum. Empiric IV antibiotics are been started. The best next step?

A

Urgent operation, not bedside drainage or placement of wound vac or monitoring. He is septic.

Deep sternal wound infection (DSWI) and mediastinitis carry high morbidity and mortality. Fortunately, DSWI occurs in only 0.2-0.5% of patients following sternotomy for cardiac surgery. Comorbidities such as diabetes, COPD, smoking, steroid use, and obesity increase risks for DSWI. Technical factors such as prolonged cardiopulmonary bypass time, increased bleeding, and sternal malunion or dehiscence further increase risk.

Superficial sternal wound infections may be managed with antibiotics and local incision and drainage. When DSWI is identified, however, surgical debridement is required. In cases of mediastinitis and sepsis, urgent operation is required. Surgical debridement should remove all necrotic and devitalized tissue, and abscess cavities should be irrigated and drained. Frequently, the sternum cannot be closed due to tissue loss or residual contamination. The open median sternotomy can be managed with sternal dressings or with negative pressure wound therapy. VAC dressings have been associated with decreased times to wound closure and may reduce overall mortality. Plastic surgery consultation can be helpful to plan reconstruction, as a variety of muscle and/or composite flaps may be required to achieve definitive wound closure.

153
Q

A 76-year-old man with hypertension and stage 3 chronic renal disease underwent surgical aortic valve replacement. His postoperative course was complicated by perioperative coagulopathy and transfusion of multiple blood products was required. Bleeding resolved within 24 hours. Now (postoperative day 5) he complains of progressive dyspnea at rest over the past several hours. Current findings include: blood pressure 99/56 mmHg, mean arterial pressure 67 mmHg, heart rate 106/min, respiratory rate 23/min, and central venous pressure 16 mmHg. His urine output was 15 and 10 mL/hr during the last 2 hours.

What happened, and what do you do?

A

This scenario raises suspicion of delayed pericardial tamponade after open cardiac surgery. The patient had valve surgery, postoperative coagulopathy, bleeding and multiple transfusions. His baseline renal dysfunction implies impaired platelet function. The findings of dyspnea, tachycardia, hypotension, oliguria, elevated central venous pressure are non-specific, but further diagnostic measures are warranted.

Echocardiography offers rapid, reliable assessment of tamponade. A transthoracic study should be attempted first. (Not CXR, TEE, or CT)

TEE is more sensitive, specific and suitable to demonstrate regional tamponade, a common finding in delayed postoperative pericardial tamponade. However, TEE is also invasive, requires sedation/anesthesia with a risk of aspiration, and it could potentially increase hemodynamic instability. Therefore, it is not the best first exam. A CT scan could also be useful, but it may introduce a delay and requires transport from the controlled environment of the intensive care unit to a remote location. A portable chest radiograph will confirm a widened mediastinum but this does not confirm tamponade.

154
Q

A 74-year-old woman underwent surgical aortic valve replacement. On postoperative day 5 small areas of skin necrosis on both hands and feet are noted. She has mild peripheral edema and adequate capillary refill. Platelets are 149 from 322 preop.
What is happening and what do you do?

A

The clinical scenario presents typical findings and initial diagnostic studies of heparin-induced thrombocytopenia (HIT) in a postoperative cardiac surgical patient. The security of this diagnosis is high (4T score, 6-8/8). An uncontrolled prothrombotic syndrome is causing skin necrosis, and it is imperative to remove heparin and order additional lab studies. Importantly, while pursuing diagnostic confirmation, immediate anticoagulation with an alternate agent is necessary, even in the absence of thrombosis. Argatroban is the recommended agent in patients with renal impairment (the bolus dose is omitted and infusion is started at 0.5-1.2 μg/kg/min.

Again, anticoagulation should not be delayed pending confirmatory tests (such as platelet-factor 4/heparin IgG immunoassay and serotonin release assay). There is no need to confirm skin necrosis with biopsy. The use of warfarin can predispose to further thrombosis, venous gangrene, and limb loss due to low protein C levels. Therefore, warfarin should not be started until after platelet count is stable and > 150 x 103 for at least 48 hours. In this case, warfarin also should overlap therapeutic argatroban coverage for at least 5 days.

155
Q

A 60-year-old man with advanced systolic, non-ischemic heart failure was admitted to the ICU. Pneumonia led to cardiac decompensation and he is on support with dobutamine and an intra-aortic balloon pump. His ICU course is complicated by progressive lethargy leading to stupor. A current physical examination shows generalized edema and adequate capillary refill. Na 119.
What is your assessment?
What is the best next step?

A

The clinical scenario describes acute, symptomatic, severe hyponatremia in a patient in acute, decompensated heart failure without cardiogenic shock. The lab data show hypervolemic, hypotonic hyponatremia. The hyponatremia (Na 119 mEq/L) is hypotonic as the serum osmolality is <280, and urine osmolality >100 suggest impaired renal capacity. The history of heart failure gives a clue to the overall edematous state, which contributes to the hypervolemia. Patients with heart failure frequently develop chronic hypervolemic hyponatremia (Na <135). However, in this case the presence of an active infection promotes the development of acute on chronic hyponatremia that is severe enough to cause neurologic manifestations.

The presence of neurologic symptoms provides a clue of the hyponatremia severity and acuity. Neurological symptoms depend on the magnitude and rate of development of hyponatremia, and usually do not manifest until the plasma Na <125. When Na falls below 120, confusion, agitation, lethargy and stupor may occur. Patients are at risk for more severe symptoms such as coma and seizure when Na drops below 110. The rate at which hyponatremia develops also effects morbidity and symptoms. Severe hyponatremia that develops in less than 24 hours is associated with more severe neurological impairment and mortality. The priority in this situation is the thoughtful correction of plasma Na. Symptoms should resolve, but osmotic demyelination syndrome must be avoided by controlling the rate of correction.

Each of the presented options represents a different approach to manage hyponatremia. Administering normal saline solution is inefficient for correction of plasma sodium. One liter of saline will increase plasma Na ~ 1 mEq/L, which will likely worsen volume overload in the described scenario. Giving furosemide might correct plasma Na too rapidly, putting this patient at risk for further neurologic insult. Correcting plasma Na by free water restriction and salt restriction will not be expeditious enough to resolve neurologic symptoms and prevent further insult.

This patient almost certainly has a component of chronic hyponatremia (>48 hrs duration) from chronic heart failure. Cautious, sole administration of 3% NaCl offers advantages given the volume status, the severity of hyponatremia and its associated neurological symptoms. The plasma sodium concentration should be corrected at a maximum rate of 0.5 mEq/(L*hr) with a goal of no more than 8-10 mEq/L improvement over the first 24 hours. Rapid overcorrection of hyponatremia can lead to osmotic demyelination syndrome that is associated with delayed neurological deficits such as dysarthria, dysphagia, gait, and behavioral disturbances and even coma.

156
Q

A 70-year-old woman underwent a mitral valve repair 2 days ago. Her initial postoperative course was unremarkable. She was extubated the evening of postoperative day 1 and advanced to a clear liquid diet. However, over the past 24 hours she has developed a fever to 39.0 C, elevated WBC count and requires reintubation. She is also tachycardic to 115 beats per minute. The CXR reveals blunting of the left costophrenic angle and a small amount of retrocardiac pneumomediastinum. The left-sided chest tube is draining turbid and cloudy fluid.
What is the diagnosis and management?

A

This patient’s symptoms are consistent with an esophageal perforation. Pneumomediastinum can only arise from an injury to the trachea or esophagus. While both structures were manipulated (trachea at intubation and esophagus during transesophageal echocardiography [TEE]), the delayed presentation and the accompanying sepsis make the esophagus the most likely source.

TEE-related esophageal perforations are rare (<1%) but serious morbidity and mortality follow if this complication is not recognized and treated adequately. Evaluation of the extent of the injury is mandated. A CT esophogram is the most sensitive study for identifying an esophageal perforation and will guide therapeutic decision making. Management of most esophageal perforations can be determined with a combination of the patients clinical picture and a CT esophogram. A fluroscopic esophogram is more specific for the location of the injury but less sensitive. The insulflation of an esophagogastroduodenoscopy can make a contained esophageal perforation worse/uncontained and may not show well the severity of the injury. If an ischemic injury or thermal injury to the esophagus is suspected, an EGD is useful. If the injury is a small size then additional drainage and observation may be appropriate. However, for a larger injury immediate operative repair is indicated. Trans-esophageal vac drainage or esophageal stenting may be an option, particularly if the patient’s condition worsens rapidly.

Obtaining cultures, antibiotic administration, and nutritional support will be parts of this patient’s management, but the clinical description suggests more serious trouble than hospital-acquired pneumonia or low-grade sepsis. Suspicion of a severe esophageal injury should lead to immediate evaluation and aggressive treatment.

157
Q

A 40-year-old woman underwent surgery on cardiopulmonary bypass for constrictive pericarditis. She was otherwise healthy except for depression for which she took sertraline (an SSRI). On arrival in the ICU she was hypotensive with no evidence of mediastinal bleeding. Her central venous pressure was 15 mmHg, pulmonary capillary wedge pressure 20 mmHg, and cardiac index 5.2 L/min/m2. Intravenous vasopressin was added to norepinephrine with little improvement. Her blood pressure decreased to 80/25 mmHg. Bedside echocardiography showed normal right ventricular function, hyperdynamic left ventricular function, and no pericardial effusion. Intravenous methylene blue administration resulted in improved hemodynamics. However, 4 hours later she now has fever to 41.1oC, hypertension, and hyper-reflexia.
What happened?

A

In patients actively receiving or who have received selective serotonin reuptake inhibitors (SSRIs), methylene blue administration can cause catastrophic acute serotonin syndrome. The primary differential diagnosis includes malignant hyperthermia and neuroleptic malignant syndrome. Serotonin syndrome should be considered in patients with high fever, hyper-reflexia, tremor or clonus, hypertension, and mydriasis. It can be fatal, resulting in rhabdomyolysis, renal failure, hepatic failure, and acidosis. Treatment is with the serotonin antagonist cyproheptadine, which blocks serotonin production, along with supportive measures. Most practitioners also advocate using dantrolene for its antipyretic effect.

Profound vasoplegia can occur after cardiopulmonary bypass. It is more common in patients taking ACE inhibitors, after left ventricular assist device/ECMO implantation, and in those with pre-existing hepatic impairment (as is often the case in chronic right or left heart failure). Vasoplegia syndrome is generally first treated with alpha-agonists (neosynephrine, norepinephrine) and vasopressin. Refractory cases may respond to IV methylene blue. This acts as an inhibitor of nitric oxide synthase and thus inhibits guanylate cylase, which decreases cGMP, which decreases vasodilation. Methylene blue also acts as a monoamine oxidase inhibitor and can increase postsynaptic serotonin levels.

158
Q

A 42-year-old-women is at postoperative day 5 from mechanical aortic valve replacement. Her temporary pacing wires and mediastinal drainage tubes were removed on postoperative day 3. Last night she became acutely unstable and developed complete heart block, for which transcutaneous pacing is initiated. After transfer to the ICU a transvenous pacing pulmonary artery (PA) catheter is placed. Shortly after that she becomes tachypneic and is gasping for air. Her central venous pressure and pulmonary artery pressures show sudden elevations in filling pressures. Auscultation reveals a “splashing” noise.
What happened?
The best next step is?

A

Large venous air emboli manifest with sudden chest pain, bradyarrhythmias, and acute right ventricular outflow obstruction that leads to right-sided heart failure, cardiogenic shock, and circulatory arrest.

Treatment is aimed to prevent further entry of air (check all pumps, IV lines, connections and stopcock positions) and to support cardiorespiratory function until the intracorporeal air is resorbed. Intubation and ventilation with 100% oxygen should be done, and needed inotropic support should be titrated to maintain cardiac output. Placing the patient in Trendelenburg and left lateral decubitus position may minimize further air entry and traps the air in the apex of the right ventricle away from the right ventricular outflow tract. Aspiration of air from the right atrium and ventricle is possibly the best treatment to improve hemodynamics immediately with up to 60% success rate. If done early, hyperbaric oxygen therapy may also be beneficial as it causes compression of existing air by establishing a high diffusion gradient.

159
Q

A 67-year-old underwent AVR/MVR for rheumatic valvular heart disease. Preoperative PA systolic pressure was 72. She is now in the ICU on postoperative day one with the following hemodynamic parameters:
HR 90, BP 110/60, CVP 12, PAP 45/20 on epinephrine 0.05 micrograms/(kg x min) with a CI of 2.3.
Her chest radiograph is shown. The patient was just now noted to have coughed up about 20 mL of bright red blood.

What happened?
What do you do?

A

Pulmonary artery branch rupture by a PA catheter is a rare event. The incidence of this complication is much less than 1/1000 cases, but it is an emergency associated with a 50% mortality. A pulmonary artery branch can be ruptured by balloon inflation by perforation by the catheter by the tip. In either case, however, the catheter tip has been advanced too far into the pulmonary parenchyma (as shown in the figure). Because of the high morbidity and mortality associated with this complication, after floating a PA catheter many cardiac programs prohibit any subsequent inflation of the balloon to determine the wedge pressure, except in unusual circumstances. The PA diastolic pressure is not a substitute for PA wedge pressure, but it can be used as a measure of LV preload, and trends provide good insight into the LV volume status.

Immediate recognition of catheter-induced PA rupture is essential to avoid overwhelming hemoptysis and hypoxia. Optimal immediate treatment can differ depending on the resources that can be brought to bear. In the ICU good choices include positioning with the “good” lung up, reversal of any anticoagulation, determining the site with bronchoscopy, and protecting the unaffected lung (placing a double lumen endotracheal tube or an occluding intrabronchial catheter). Pulmonary arteriography and embolization is highly effective and is recommended if bleeding does not stop promptly.

Maintaining a clear airway to the uninvolved lung is the primary immediate goal, and lung resection should be considered a last resort. Suture of a ruptured PA is rarely feasible in the acute setting, but a late-appearing pseudoaneurysm should be repaired.

160
Q

A 61-year-old man presented with progressive dyspnea, orthopnea, and 2+ lower extremity edema. His workup revealed severe mitral stenosis and severe tricuspid regurgitation. Mitral valve replacement and tricuspid valve annuloplasty were done 3 days ago. Heparin was started for persistent rapid atrial fibrillation and an amiodarone bolus dose of 150 mg x 2 was followed by a continuous IV drip. He received a total of 1,350 mg of IV amiodarone over a 24-hour period. The following day he appeared jaundiced and his serum aminotransferase and bilirubin levels were markedly elevated. His alkaline phosphatase level, however, was normal.

Assessment/plan?

A

Liver injury from amiodarone is uncommon but not rare.

Treatment is supportive, and stopping amiodarone usually results in prompt resolution.

Amiodarone is an iodinated benzofuran derivative that is a structural analogue of thyroid hormone. It has excellent antiarrhythmic effects. It is highly effective for suppressing ventricular arrhythmias and to maintain sinus rhythm in patients with atrial fibrillation. Elevations of serum hepatic enzymes are found in 15-50% of patients treated with amiodarone. The liver injury occurs more frequently with high doses and prolonged therapy. Patients develop symptoms of fatigue, nausea, and weight loss with or without jaundice, and moderate elevations in serum aminotransferase and alkaline phosphatase levels are typical. The injury resembles alcoholic liver disease clinically. When amiodarone is given intravenously in high doses, acute liver injury can occur within a day of infusion. Marked elevation in serum transaminases are found.

161
Q

A 56-year-old woman underwent an uncomplicated mitral valve repair for severe regurgitation. On postoperative day 3 she developed atrial fibrillation with a ventricular rate in the 140’s. Her blood pressure was 115/75 mmHg and she denied any symptoms. Oral metoprolol and IV amiodarone were administered, yet after 12 hours she remains in atrial fibrillation with a ventricular rate in the 120’s and with a stable blood pressure.

What are options for managment?

A

Failing to address a ventricular rate over 120 is not appropriate.

The described patient went into stable rapid atrial fibrillation on postoperative day 2, and initial attempts to restore a normal rate and rhythm failed.
* Attempting synchronized electrical cardioversion in this setting is reasonable, although moderate sedation is required.
* Using diltiazem for rate control while achieving anticoagulation with unfractionated heparin is also acceptable, although transition to warfarin will be necessary for longer term anticoagulation.
* Amiodarone and ibutilide are independently effective for chemical cardioversion and both have been shown to increase the efficacy of electrical cardioversion. A full loading dose of amiodarone (nearly 10 grams) requires a week or longer to achieve. Clinical circumstances and potential side effects of both amiodarone and ibutilide must be considered before use. Providing anticoagulation with heparin and/or warfarin is appropriate for persistent atrial fibrillation.
* It is also imperative to achieve ventricular rate control between 60-100/min.

Atrial fibrillation is one of the most common complications following cardiac surgery, and occurs in 25%-35% of postoperative patients. Many general risk factors have been cited, such as advanced age, a history of arrhythmias, lung disease, valve surgery, and not receiving postoperative beta-blockers. While there is no uniformly accepted medical therapy, early and aggressive treatment is prudent to reduce the risk of stroke. The duration of early postoperative atrial fibrillation is usually less than 6 weeks, but current recommendations favor anticoagulation with warfarin as stroke prophylaxis during that time.

162
Q

An 81-year-old is undergoing preoperative evaluation for elective coronary artery bypass surgery. They have a history of atrial fibrillation treated with chronic anticoagulation, hypertension, and diabetes. They have no history of dementia, however the accompanying daughter states they are occasionally forgetful. The best method of assessing a patient’s cognitive abilities in the preoperative setting is?

A

Mini-Cog

The prevalence of dementia increases exponentially with increasing age older than 65 years. Pre-existing cognitive impairment strongly predicts postoperative delirium, which is associated with worse surgical outcomes including longer hospital stays, increased risk of perioperative mortality, and postoperative functional decline. Of these tests, the Mini-Cog is a preferred test that can be used during an in-person clinic visit in the preoperative setting. The Fried Frailty Assessment screens for 3 or more of the following criteria: unintentional weight loss (10 lbs in past year), self-reported exhaustion, weakness (grip strength), slow walking speed, and low physical activity. It does not examine for cognition. The Alcohol Use Disorders Identification Test (AUDIT-C) is a screen for heavy alcohol consumption - not directly for cognition. The Geriatric Depression Scale (GDS) was designed specifically to rate depression in the older adult, not cognition.

163
Q

Eight days after a complicated aortic valve replacement procedure, the patient returns to the intensive care unit with confusion, fevers, cough, and new pulmonary infiltrates on CXR. The postoperative course has been complicated by a small cerebrovascular accident, mild renal failure, thrombocytopenia, and new-onset atrial fibrillation. Blood cultures are pending; vital signs include temperature 40.1oC, heart rate 123 beats per minute, respiratory rate 28 breaths/min, oxygen saturation 98% on 4 L oxygen by nasal cannula, and mean arterial pressure 64 mmHg. The best next step is?

A

The patient has signs of early sepsis. Multiple studies regarding the role of steroids in sepsis have been conducted, but a definitive answer has not been elucidated. The 2012 Surviving Sepsis Campaign guidelines do not recommend using steroids if “adequate fluid resuscitation and vasopressor therapy are able to restore hemodynamic stability.” Otherwise it is suggested to use IV hydrocortisone 200 mg per day (grade 2C recommendation). Animal studies of septic shock in the 1970s demonstrated improved response to steroids when given in combination with antibiotics. Timing of antibiotic administration to patients with sepsis demonstrated improved survival with earlier administration.

Treating early shock with Goal Directed Resuscitation using invasive hemodynamic monitoring has been called into question in recent studies. There is a 1A level recommendation to avoiding routine use of pulmonary artery catheters in ARDS and acute lung injury. Swallowing dysfunction following CVA can lead to complications such as pneumonia and sepsis, so caution is advised when restarting oral intake. A formal swallowing study workup should be performed after the patient has been stabilized. The need for synchronized cardioversion of atrial fibrillation depends on the adequacy of perfusion, the ventricular response rate, stroke concerns, other factors that promote this arrhythmia (especially drugs), and the likelihood that sinus rhythm can be maintained.

164
Q

A 55-year-old man is seen 3 weeks following his hospitalization for coronary bypass surgery. He describes “clicking” in his chest with movement, and there is bloody drainage on his clothing. CT imaging confirms a deep sternal wound infection.
How do you manage?

A

Treatment of sternal wounds depends on the extent.

Optimal treatment usually requires a combination of surgical debridement and antimicrobial therapy. Debridement of all nonviable or infected tissue is followed either by immediate primary closure or by open wound care for an interval and then delayed closure with or without muscle flaps to import healthy tissue and a new blood supply. Antimicrobial therapy alone is rarely successful at eradicating a deep sternal wound infection (DSWI). A lengthy delay of surgical treatment is counterproductive.

For those patients with an open wound following debridement a VAC device can be used prior to final closure. Clinical benefits of negative pressure wound therapy can include decreased hospital length of stay, decreased rates of reinfection, and perhaps even decreased early mortality. However, VAC dressings changed every 48 hours are not a substitute for surgical debridement of all nonviable tissue.

A superficial sternal wound infection that does not involve the sternum can be treated with local wound care. A full sternectomy and muscle flap closure as an initial procedure is overly aggressive for most instances of DSWI.

165
Q

What is the most validated indicator of weaning success in a ventilated patient?
What are other indicators for weaning success depending on the situation?

A

Indicators of weaning success have been tested primarily in patients who have been ventilated for short periods of time. For patients requiring longer periods (>72 hours) of support it is more difficult to estimate the likelihood of successful extubation.

When the **rapid shallow breathing index (RSBI) ** is >105 breaths/(L x min) up to 95% of the attempts to discontinue mechanical ventilation will fail. In general, the lower the RSBI, the more likely a patient is to be successfully extubated.

The same is true for respiratory rate, with >40 breaths/min being an indicator of increased work of breathing.

The strength of the diaphragm and respiratory muscles can be evaluated by having a patient exhale to a residual lung volume and then inhale as forcefully as possible. This pressure generated is called a maximum inspiratory pressure (MIP) or negative inspiratory force (NIF). When this value -30 cm H20 or less the likelihood of successful extubation is great; when it is greater than -20 cm H20, it implies poor reserve and a high likelihood of reintubation. A patient with an inadequate maximum inspiratory pressure may require less sedation or more time for paralytics from surgery to wear off.

Additionally, tidal volumes >5 L/min or >6 mL/kg of ideal body weight are a positive indicator of successful extubation. Total ventilation of 5-6 L/min is normal for an adult patient, and if after a spontaneous breathing trial the total ventilation remains <10 L/min, a positive outcome is more likely.

166
Q

A 64-year-old man is postoperative day 3 after coronary artery bypass graft surgery. He develops new onset of paroxysmal atrial flutter with a heart rate of 155 beats/minute. Temporary epicardial pacing wires are in place. Biventricular function is normal with a left ventricular ejection fraction >55%. The patient complains of sudden dizziness and is slightly confused.
What can you do?

A

The clinical scenario describes a patient post-cardiac surgery with new onset atrial flutter with a rapid ventricular response. The man is hypotensive and symptomatic. In this situation, the arrhythmia should be promptly terminated.

Since the patient has a temporary epicardial pacemaker, rapid atrial pacing provides a rapid treatment option that avoids potential hemodynamic instability induced by anesthesia or sedation. Moreover, the success rate of rapid atrial pacing for flutter exceeds 50%.
Rapid atrial pacing can be attempted before electrical cardioversion.

Direct current synchronized cardioversion has a success rate >96%, but can predispose the patient to aspiration and further hemodynamic compromise given the need for anesthesia and sedation.

Defibrillation is not indicated. He is not fibrillating.

Pharmacologic conversion with amiodarone is less effective and is not an appropriate choice when there is hemodynamic instability. If electrolytes are normal, supplemental potassium and/or magnesium is neither necessary nor recommended.

167
Q

A 37-year-old man underwent mitral valve replacement. His postoperative course has been complicated by pneumonia that progressed to adult respiratory distress syndrome. He is currently intubated and sedated. He is now afebrile with heart rate 84/min, blood pressure 124/84 mmHg, and SpO2 90%. Arterial blood gas values are: pH 7.32, pCO2 52 torr, pO2 63 torr.
Vent at 80%, PEEP 14, rate 26, TV 8 ml/kg of IBW.
What can be optimized?

A

Treat the ARDS. Drop TV to 6 ml/kg of IBW. Tolerate hypercarbia as long as pH is ok.

A cornerstone of treatment for ARDS is low tidal volume ventilation. The ARDSNet trial group demonstrated this with their landmark paper published in the New England Journal of Medicine in 2000. This study demonstrated a mortality benefit to low tidal volume ventilation, defined as tidal volume = 6 mL/kg of ideal body weight and plateau pressure < 30 cm H2O. The described patient is adequately oxygenated so neither FiO2 nor PEEP need be increased. Because the tidal volume guideline is low, alveolar and minute ventilation are also low. This often leads to mild-to-moderate hypercarbia, but it is well tolerated. In this case, the pH and pCO2 are acceptable and the ventilator rate does not need to be increased.

168
Q

The 3 main principles of informed consent?

A

disclosure (honesty when discussing risks and benefits, alternatives to treatment, physician level of experience), capacity (does this patient have the mental ability to understand what is being said?), and voluntariness (is the patient somehow being coerced into signing the form?)

169
Q

Major factors for GI bleeding in the ICU?

A

Mechanical ventilation x48 hrs, coagulopathy - both of these together create a 4% incidence of major GIB.
Other major: hx of ulceration or bleeding within the past year.
Minor: TBI or spinal injury, burn, sepsis, glucocorticoid use, ICU stay for 1 week.
PPI may be more effective than H2 blocker.
Ppx drugs may be associated with HAP.

170
Q

Can benzos be used for insomnia?

A

Older patients, their caregivers, and their providers should recognize potential harms when considering treatment strategies for insomnia. The use of hypnotics/sedation (such as benzodiazepines) has been associated with the occurrence of delirium. Use of these agents should be reserved for alcohol withdrawal symptoms/delirium tremens or severe generalized anxiety disorder unresponsive to other therapies.

171
Q

Ideally, when should nutrition be started for critically ill patients?

A

Another recent recommendation is that nutritional support be initiated earlier than prior guidelines, and within 24-48 hours in critically ill patients. In general, enteral feeding is preferred over parenteral nutrition (“if the gut works use it”) unless enteral feeds are not tolerated despite promotility agents and other maneuvers. Gastric feeding is indicated for most ICU patients, but a lower rate of infusion should be undertaken for patients at high risk for aspiration or those who have shown intolerance. Post-pyloric feeding is appropriate for patients deemed at high risk for aspiration. Literature does not support waiting for return of bowel function (bowel sounds and passing flatus or stool) prior to initiating enteral feeding. However, enteral feeding should probably be withheld in the setting of hemodynamic compromise or instability, but can be initiated or reinitiated with caution once vasoactive agents have been titrated to stable low doses or off.

Traditional visceral protein levels such as serum albumin, pre-albumin, and transferrin should not be used as indicators of the need for nutritional support for many ICU patients. The stress of illness, acutely altered metabolic profiles, and innate catabolism associated with healing all cloud interpretation of standard acute phase proteins regarding nutritional health. A nutritional risk assessment should be performed on all patients admitted to the ICU. In fact, an initial nutritional screen is recommended for all patients within 48 hours of admission to the hospital. Individuals at high nutritional risk are likely to benefit from early enteral nutritional therapy.

172
Q

A 55-year-old man underwent aortic valve replacement with a mechanical prosthetic valve. In the ICU he has frequent premature ventricular contractions and recurrent episodes of non-sustained ventricular tachycardia without hemodynamic instability. Hemodynamics and laboratory values at this time are: Blood pressure - 120/65 mmHg, Pulmonary artery pressure - 30/3 mmHg, Central venous pressure - 7 mmHg, Cardiac index - 2.8 L/min/m2, Potassium - 4.2 mEq/L, Magnesium - 2.6 mEq/L.

The best next step is?

A

The most common complication associated with PA catheters are arrhythmias and bundle branch block. These frequently occur when the catheter slips back into the right ventricle. Data for the described patient includes a PA pressure reading of 30/3 mmHg, which is almost certainly a right ventricular pressure since a minimal normal diastolic PA pressure is 8-10 mmHg. Arrhythmias provoked by a mobile catheter tip within the right ventricle may be corrected by advancing or removing the PA catheter. This patient is stable and likely won’t need the PA catheter, so it should be removed.

The most dreaded complication is PA rupture, which can occur when the catheter is advanced too far into the PA and “overwedged.” The tip of the catheter can erode and perforate a PA branch, causing sudden hemoptysis. If the catheter is too far out and the balloon is inflated to measure a wedge pressure, then catastrophic rupture of a branch PA can occur. Other complications associated with Swan-Ganz catheters include pulmonary infarction, infection, and thromboembolism. Appropriate position of the catheter can be ascertained by evaluation of the PA tracing, the insertion length, and the CXR.

173
Q

Most frequent adverse effect of IABP? What do you do?

A

IABP counterpulsation is commonly used for cardiac support in critically ill patients. The most frequent adverse effect of IABP is thrombocytopenia, which has been reported to occur in 47-82% of patients. Platelet count begins to fall immediately after IABP insertion and continues to fall through day 3-4. The mean maximum percent reduction in platelet count from baseline is usually 30-35%.

In a recent study by Roy, et al the reduction in platelet count among patients undergoing IABP was found to be independent of the use of clopidogrel and glycoprotein IIb/IIIa inhibitors. This suggests the platelet reductions result from mechanical effects of IABP. Also, the incidence of major bleeding was substantially greater among patients who developed thrombocytopenia (13.8% vs. 4.2%). However, after controlling for patient differences (particularly the duration of IABP support) thrombocytopenia was not significantly associated with major bleeding. Similarly, although in-hospital death was more common among IABP patients who developed thrombocytopenia than among those who did not (28% vs. 16%), thrombocytopenia was not an independent predictor of in hospital death. In conclusion, thrombocytopenia is a common, but generally mild adverse effect of IABP therapy. Platelet count reduction is related to mechanical factors and is not associated with major bleeding or in hospital death. Therefore, thrombocytopenia should not prompt stopping IABP support or adding important adjunctive medications.

174
Q

A 68-year-old woman with type II diabetes mellitus and morbid obesity has triple-vessel coronary artery disease. After an uncomplicated coronary artery bypass surgery including use of the left internal mammary artery, the patient is discharged. She returns on postoperative day 16 with purulent drainage from the sternal wound and wound dehiscence at the junction of the distal and middle thirds. A CT scan shows peri-sternal fluid accumulation and necrotic sternal bone edges. Methicillin-resistant Staphylococcus aureus is cultured. She undergoes extensive debridement of sternum. Wnat is the best plan going forward?

A

VAC therapy until infection is controlled evidenced by negative cultures and healthy granulation tissue. Then early surgical closure with flaps, usually pectoralis flaps.

Post-sternotomy mediastinitis (PSM) and deep sternal wound infection (DSWI) is a serious complication after cardiac surgery associated with a high mortality rate, especially in the setting of a mediastinitis. Vacuum (VAC) system has emerged as a valuable tool and effective adjunct to surgical management of DWSI. The negative pressure suction of VAC therapy enhances the local tissue blood flow, decreases edema and increases the formation of granulation tissue. Furthermore, the VAC system stabilizes the sternal edges and keeps them relatively closer together. This reduces the shear forces between the beating heart and the sternum, and may prevent the rare but fatal complication of right ventricular rupture. It also provides extra stability to the thorax, thereby helping in faster extubation. Furthermore, the use of VAC therapy rapidly allows the patients to regain mobility. This carries a significant impact on nursing care requirement and a lowered risk for deep vein thrombosis.

Various reports have shown that VAC therapy is equally effective in treating superficial and DSWI. Studies comparing the VAC therapy with conventional wound care strategies have concluded that use of VAC therapy provides lower risk of treatment failure, a significantly shorter hospital stay, faster control over the infection, and a significantly lowered short and long term mortality. Furthermore, the VAC change can be performed either at bedside or in the operating room as an adjunct to wound wash out and debridement of all the necrotic and devitalized tissue.

The level I evidence in management of sternal wound infection is limited. However, VAC therapy has been shown to be useful in the treatment of sternal osteomyelitis as a temporary wound care technique preoperatively, as the primary method of secondary wound closure in selected cases and as a technique to facilitate healing prior or after the flap reconstruction in cases complicated by reinfection. The prolonged use of VAC therapy for definitive secondary closure, as a replacement of surgical closure, has however shown less than optimal results with higher rate of recurrent wound infection. The most favorable outcomes have been achieved by using the VAC Therapy for a shorter duration followed by early surgical closure.

The decision to discontinue VAC therapy is usually guided by resolving clinical signs of infection and inflammation, negative bacterial cultures, and appearance of healthy granulation tissue. The use of pectoralis muscle flap to close the sternal defects is a preferred method. Rectus abdominis flap has been proven to be useful for sternal reconstruction, especially if the pectoralis flap is not available or does not provide sufficient soft tissue coverage. It can be lifted as a musculocutaneous flap and provides sufficient volume to fill the dead-space. The need of extensive dissection to harvest the rectus abdominis and associated risk of ventral hernia formation and uneven contour of the abdominal wall makes this as not a first choice flap. The use of omental transposition to cover sternal defects has been used less frequently than the muscular flaps. Due to rich omentum blood supply and well-developed lymphatic system, several studies conclude that transposed omtentum, either alone or in combination with muscle flaps still holds an important place in the reconstruction of sternum after DSWI.

The primary sternal approximation and rewiring is possible in selected cases with negative wound cultures and intact sternal bone. The loss of significant sternal bone edges and an ongoing wound infection in this patient makes her an unsuitable candidate for this strategy.

175
Q

A 65-year-old man with long-standing hypertension presented with a Type B aortic dissection. Both esmolol and nitroprusside were required to control mean arterial blood pressure at 70 mmHg and heart rate at 70 beats/min. Three days later he is confused and difficult to arouse. He is afebrile with a blood pressure of 124/75 mmHg, heart rate is 105 beats/min, respirations 22/min, and O2 saturation 94% on 2 L nasal cannula oxygen. Physical examination shows diffuse flushing. Stat laboratory studies show sodium 138 mEq/L, potassium 4.2 mEq/L, chloride 90 mEq/L, bicarbonate 18 mEq/L, creatinine 1.6 mg/dL, and BUN 32 mg/dL.

What happened?
The best next step is to?

A

This patient is presenting with cyanide toxicity from prolonged nitroprusside infusion. Signs of cyanide poisoning include headache, confusion progressing to obtundation, seizures, nausea, vomiting, and flushing. In this setting, nitroprusside must be discontinued, but if symptoms are severe the patient should also be treated with sodium thiosulfate often in conjunction with sodium nitrate.

Cyanide is not dialyzable, although hemodialysis can be used later to clear thiocyanate. There are case reports of using plasma exchange to treat cyanide toxicity but this is not a standard of care. Methylene blue is used to treat methemoglobinemia, not cyanide toxicity, but sodium nitrate treatment can generate a need for this drug.

176
Q

A 61-year-old man had a history of hypertension (on beta-blockers), diabetes (managed medically), chronic obstructive pulmonary disease (on a fluticasone inhaler), and severe eczema (on several topical steroid preparations). He suffered an acute type A aortic dissection and after successful emergency repair he was transferred to the ICU on low-dose epinephrine. Over the next few hours, the patient received 1.5 L of crystalloid solution and his epinephrine infusion was doubled due to hypotension. A vasopressin and norepinephrine infusion were added with modest response. An ECG showed no ischemic changes and a new chest radiograph is shows only expected changes. He is presently afebrile, his heart rate = 85, blood pressure = 75/40 mmHg, and oxygen saturation is 100% on mechanical ventilation. CVP = 10, PA pressure = 35/15, and PCW = 10. He has no jugular venous distention but breath sounds are course bilaterally. His abdomen is warm and soft and all pulses are adequate. The chest tubes are draining 30-40 mL of serosanguinous fluid per hour. Labs include: hemoglobin 9.2, Na 130, K 5.4, ionized Ca 1.2, BUN 35, creatinine 1.1.
What is happening?
The best management is to?

A

This patient is exhibiting signs of acute adrenal insufficiency with refractory shock, hyponatremia, and hyperkalemia. Patients on chronic steroid therapy may have a suppressed hypothalamic-pituitary-adrenal (HPA) axis, and they may not be able to mount an appropriate adrenal stress response (adrenal insufficiency). Therefore, perioperative glucocorticoid administration is recommended in patients receiving the equivalent of 20 mg or more of prednisone per day for more than 3 weeks. Patients taking 5-20 mg of prednisone per day and those that have discontinued the use of steroids over the previous year should undergo preoperative evaluation of the HPA axis if possible or, alternatively, should have prophylactic perioperative administration of steroids. Topical and inhaled glucocorticoids (such as fluticasone) also have the potential to suppress the HPA axis. Perioperative steroid replacement therapy for these patients is needed if they develop any signs of adrenal insufficiency (as did the described patient). The use of etomidate has also been associated with relative adrenal insufficiency in patients after cardiac surgery.

In addition to further intravenous fluid resuscitation, this patient should undergo empiric steroid supplementation while waiting for the results of a cortisol stimulation test. A cortisol stimulation test is performed by administering 250 μg of synthetic corticotropin (cosyntropin) intravenously and measuring the concentration of cortisol at 0, 30, and 60 minutes. A normal response is an increase in cortisol concentration after 30 or 60 minutes to a peak of 18-20 μg/dL or more. IV hydrocortisone is the glucocorticoid most commonly used for steroid supplementation but it can interfere with the cortisol stimulation test. Therefore, dexamethasone (4 mg intravenous bolus) should be used initially in this patient. A good hemodynamic response should be seen within a few hours of steroid treatment. After the corticotropin test is done, the patient should be treated with 100 mg of IV hydrocortisone every 8 hours. This can then be tapered over several days until the patient is beyond the acute stress period and his regular steroids are restarted.

An echocardiogram can be useful if there is a suspicion of pericardial tamponade or cardiogenic shock. Immediately after surgery, however, a transesophageal study is superior to a precordial exam. The relatively low central venous pressure and pulmonary capillary wedge pressure in this patient make these diagnoses unlikely. With no concerns of tamponade or bleeding there is no indication for mediastinal re-exploration. Adding a vasopressor will not improve this patient’s blood pressure unless steroids are given as well. The indicated calcium level is normal. Vasoplegia syndrome is another consideration in this case scenario, but this syndrome usually features severe hypotension (<50 mmHg systolic) despite moderate norepinephrine infusion.

177
Q

Three days ago a 53-year-old man had a dual chamber pacemaker/ICD placed for ischemic cardiomyopathy. His history included 4-vessel CABG years ago, and his renal function is normal. He is being treated for paroxysmal atrial fibrillation while in the hospital. After 2 doses of sotalol (80 mg bid), his blood pressure is stable and his heart rate is 69. However, the corrected QT interval changed from 440 to 600. The most appropriate intervention is to

A

The patient is being loaded with a potent Class III antiarrhythmic, sotalol, which is also a beta-blocker. Class III antiarrhythmics, which also includes dofetilide, ibutilide, and amiodarone, work by effecting potassium ion efflux, which prolongs phase 3 of the cardiac action potential. This results in prolongation of the QT interval, which introduces a risk of torsades de pointes (polymorphic ventricular tachycardia). Close ECG monitoring is necessary when administering class III antiarrhythmics and if the corrected QT interval* (QTc) is prolonged to greater than 500 msec, then an adjustment in therapy is needed. Other causes of QT prolongation include class IA drugs (quinidine, procainamide, disopyramide), class IV drugs (verapamil cisapride, hypokalemia, hypomagnessemia), and familial QT prolongation syndromes.

While amiodarone is less likely than other Class III antiarrhythmics to cause torsades de pointes (1% or less), it is still a risk factor and switching will not shorten this patient’s prolonged QTc. IV magnesium is appropriate therapy for both treating and preventing torsades de points. This is especially true if the magnesium level is low, but treatment will not affect the QTc immediately. Potassium should be given in most cases along with magnesium to keep the potassium level in the high normal range. Continuing sotalol in the presence of a prolonged QTc is ill-advised. Isoproterenol is contraindicated in the presence of congenital long QT syndromes and severe ischemic heart disease, but in other circumstances it can be used to temporarily increase the heart rate if external pacing is not available. In the described scenario the pacemaker rate should be increased, which will shorten the QTc by enhancing repolarization of potassium currents. Increasing the heart rate is a highly effective way of preventing torsades de pointes.

  • Bazett’s formula for calculating the corrected QT interval: QTc = (QT interval in msec) / (square root of the R-R interval in sec.
178
Q

Three weeks after coronary artery bypass graft (CABG) a 67-year-old man requires readmission because of severe lower extremity edema and a weight gain of 10 kilograms over the last 3 days. His vitals signs are stable. His wife states that he is not himself and he is lethargic and confused.

What is the most likely electrolyte derangement in this man?

A

This elderly man is most likely suffering from symptomatic hyponatremia with encephalopathy, which is a medical emergency. Hyponatremia is defined as a serum sodium <135 mEq/L. Under normal situations the kidneys are able to maintain serum sodium of 135-145 mEq/L, filtering up to 15 L/day via sodium and free water exchange. Hyponatremia usually develops when underlying conditions impair the kidney’s ability to excrete free water. Postoperative hyponatremia is a relatively common clinical problem occurring in 1% of postoperative CABG patients, with symptoms occurring in 20% of them. Postoperative patients develop hyponatremia due to a combination of non-osmotic stimuli for ADH release, such as subclinical volume depletion, pain, nausea, stress, edema-forming conditions and administration of hypotonic fluids.

The presence and severity of symptoms are associated with the level of serum sodium. When sodium levels in blood become too low (Na <115 mEq), excess water enters cells and causes the cells to swell. This is especially dangerous in the brain and can lead to hyponatremic encephalopathy with headache, nausea, vomiting and confusion, seizures, respiratory arrest and non-cardiogenic pulmonary edema. In the most severe cases it can lead to tentorial herniation with brain stem compression, respiratory arrest and death.

Hyponatremia is prevented by limiting hypotonic fluids postoperatively, using isotonic saline unless otherwise clinically indicated. The serum sodium should be measured daily in patients receiving continuous parenteral fluids. In asymptomatic patients, low sodium is treated by free water restriction and attention to underlying disorder(s). Correction with hypertonic saline is not necessary and potentially harmful if there are no neurological abnormalities. Encephalopathy, on the other hand, indicates a medical emergency that should be treated with hypertonic saline (3%, 515 mEq/L) until the patient is symptom free or until serum sodium is increased by 20 mEq/L in the initial 48 hours of therapy. In general, the serum sodium should be corrected only to 125-130 mEq/L.

Acute symptoms from low levels of magnesium, calcium, potassium or phosphate are extremely uncommon compared to hyponatremia. Symptoms of hypomagnesemia and hypocalcemia can be very similar to hyponatremia, including weakness, muscle cramps, cardiac arrhythmia, and increased irritability of the nervous system disorders as well as confusion, disorientation, and hallucinations. Hypophosphatemia can cause muscle dysfunction, respiratory depression, mental status changes, white cell dysfunction, and hemolytic anemia. Hypokalemia can also cause muscle weakness, EKG changes, and hyporeflexia.

179
Q

A 43-year-old, 220 pound woman in multisystem organ failure was placed on venous-venous extracorporeal membrane oxygenator 3 days ago secondary to community acquired pneumonia. She has been weaned off all vasoactive drugs, but is still requiring mechanical ventilation. Continuous isovolumetric enteric feeding is through a post pyloric Dobhoff tube. Her most recent set of electrolytes are: Na 160, K 4.1, Cl 115, CO2 24, BUN 16, Creat 1.1, albumin 4 and prealbumin is 25. In addition, she is currently getting 1 L free water for insensible loses. Which of the following is the best way to correct her hypernatremia?

A

While this patient is very sick she is meeting her nutritional goals, as her albumin and prealbumin are both in the normal range. Therefore there is little cause to change her route or type of feeds. She needs additional free water. Hypernatremia is a common issue in ICU patients. It is defined as a serum sodium level of >145 mEq/L. It is usually not caused by an excess of sodium, but rather a relative deficit of free water in the body. The normal physiologic response to an increase in sodium is thirst, but many ICU patients are unable to communicate this. Clinical symptoms can be lethargy, weakness, irritability, neuromuscular excitability, and/or edema. At higher sodium levels (i.e., >160) seizures and coma can occur. There are 3 types of hypernatremia: hypovolemic, euvolemic, and hypervolemic. The most common type is hypovolemic and is usually due to an inadequate intake of water. In euvolemic there is excessive excretion of water as a result of diabetes insipidus, which is due to either an inadequate amount of vaspressin secreted from the pineal gland or an impaired response to the hormone by the kidneys. Hypervolemic hypernatremia is relatively uncommon except in the hospital setting, and is usually caused by clinical interventions. The free water deficit of hypernatremia is calculated by initially determining total body water, using the formula TBW = weight (kg) x a correction factor. The correction factor is different for children and nonelderly men (0.6), for women and elderly men (0.5), and for elderly women (0.45). Another calculation predicts the change in serum sodium from infusion of one liter of different solutions: Change in serum Na+ = (infusate Na+ - serum Na+) ÷ (TBW + 1) Water can be replaced orally or intravenously, but since water is a hypo-osmotic fluid it must be administered with dextrose or saline if through an intravenous route. Correction must be done slowly to avoid rapid fluid shifts that can cause cerebral edema. Treatment entails not only replacing the free water but finding and correcting the underlying problem. In long-standing hypernatremia or cases of unknown duration, correcting the sodium at a maximal rate of 0.5 mEq/(L*hour) prevents cerebral edema and convulsions. The goal should be 145 mEq/L. The preferred route for fluid administration is orally or by feeding tube. If IV correction is required, then 5% dextrose, 0.2% NaCl, or 0.45% NaCl can be used. - 5% dextrose in water (D5W): 0 mEq/L Na- 0.2% sodium chloride in 5% dextrose in water (D51/4 NS): 34 mEq/L Na- 0.45% sodium chloride in water (0.5 NS): 77 mEq/L Na- Ringer’s lactate solution: 130 mEq/L Na- 0.9% sodium chloride in water (0.9NS): 154 mEq/L Putting it all together: a 43-year-old 100 kg woman with serum Na of 160 mEq has a TBW of 100 x 0.5 = 50 L. Using D5W, retention of 1 L will reduce her serum sodium by (0 - 160) ÷ (50 + 1) = -3.14 mEq. The goal is to reduce her serum sodium by no more than 10 mEq/L in a 24-hour period. so (10 ÷ 3.14) = 3.18 L of solution is required. About 1 L or 40 mL/hour will be added for obligatory and insensible water loss to make a total of up to 4.18 L of D5 W over 24 hours, or 174 mL/h. Continuing the infusion at this rate for 36 hours should safely arrive at a final serum sodium of 145 mEq/L.

180
Q

A 25-year-old woman had a successful repair of complete atrioventricular septal defect and insertion of an epicardial pacemaker as a child. Yesterday she underwent redo sternotomy for repair of right atrioventricular valve regurgitation and a pacemaker generator change. She was extubated a few hours after surgery. This morning (postoperative day 1) her chest tube output remained bloody and her INR was 1.4. She was transfused 2 units of fresh frozen plasma. Two hours later she developed progressive respiratory distress and required intubation. Current vital signs include temperature = 37.0°C, heart rate = 110/min, and blood pressure is 105/70. Her O2 saturation is 97% on FiO2 = 0.5, PEEP of 7 cmH2O, and appropriate minute ventilation with a tidal volume of 5 mL/kg. Pink frothy secretions are seen within the endotracheal tube. She has no jugular venous distention and the rest of her exam is unremarkable. The portable chest radiograph shows bilateral patchy infiltrates w/o effusion or mediastinal enlargement. Brain natriuretic peptide levels are normal. The most likely etiology of the patient’s clinical condition is

A

This patient has developed transfusion-related acute lung injury (TRALI). TRALI is defined as the development of acute lung injury during or within 6 hours after transfusion of one or more units of blood or blood products. TRALI occurs in 1/1,000-5,000 units transfused and is the most common cause of transfusion-related mortality. The mortality is estimated to be 5%-8% although some reports of TRALI after cardiac surgery have shown a mortality as high as 13%. The etiology is unclear but it is believed to be secondary to antibodies or factors that accumulate during storage of blood products in the setting of a predisposing inflammatory condition in the host. Identified risk factors include prolonged storage of blood products, use of fresh frozen plasma, recent surgery, emergency cardiac surgery, duration of cardiopulmonary bypass, sepsis, shock, burns, alcohol use, and smoking, among others. Administration of plasma or whole blood from female donors, in particular multiparous women, has been implicated with TRALI, probably related to alloimmunization from pregnancy. HLA antibodies are found in more than 30% of women with four or more pregnancies compared to just 1% in male patients without a prior blood transfusion.

TRALI can manifest as fever, tachycardia, and tachypnea shortly after a blood transfusion. Patients develop respiratory failure and approximately 70% of them require mechanical ventilation. Frothy secretions indicative of pulmonary edema are common. The chest radiograph, as in this patient, shows bilateral patchy alveolar infiltrates. Right ventricular failure and massive pulmonary embolism (also right ventricular failure) do not feature signs and symptoms of pulmonary edema as in this case. Adverse reactions to protamine include histamine-related hypotension, anaphylaxis (IgG, IgE), anaphylactoid reactions (IgG, compliment activation), “catastrophic pulmonary hypertension” (thromboxane A2 mediated), and “delayed noncardiogenic pulmonary edema.” The latter diagnosis is an uncertain entity which has most often been indentified in postoperative situations where transfusion has also been necessary. Each of the protamine-associated diagnoses manifests either immediately or within less than one hour of protamine administration, which is inconsistent with the timeline of the described clinical scenario.

Management of TRALI is supportive with a protective lung ventilation strategy (low tidal volumes and plateau pressures). Resolution tends to be rapid and most patients can be extubated within 48 hours. Radiographic changes resolve within 4 days. If the patient is being transfused at the time of onset of symptoms, the transfusion should be stopped immediately. The blood bank should be notified in all cases of TRALI in order to allow serologic analysis for leukocyte antibodies in the donor and recipient. Diuresis is only indicated when there is evidence of fluid overload such as elevated jugular venous pressure or elevated brain natriuretic peptide. Fluid administration may be necessary in patients with hypotension. There is no clear evidence that steroids improve outcomes. Antibiotics are not indicated for TRALI. The vital signs of this patient are acceptable on conventional mechanical ventilator support, so ECMO is not indicated.

181
Q

Nine days ago 45-year-old woman underwent redo sternotomy and aortic root replacement with a homograft. Platelet counts have been trending down from a baseline of 200,000/mm3 and are now 55,000. She is getting 81 mg ASA per day, linezolid for pneumonia, and amiodarone for postoperative atrial fibrillation. She has stable vital signs and hemoglobin levels. An anti-platelet antigen and function tests were negative for immune heparin-induced thrombocytopenia.

What is happening?
What do you do?

A

Linezolid side effect. Switch the abx.

Linezolid is an oxazalidinone class bacterostatic agent with near 100% oral bioavailability. It is typically used against gram positive organisms and is particularly useful in treating drug resistant bacteria. However, it has been associated with thrombocytopenia. During phase III trials, 2.4% of patients developed reversible thrombocytopenia. A matched cohort study at the Veterans’ Affairs system of 502 patients compared linezolid to vancomycin and showed 17% of patients getting linezolid had a decline in platelet count greater than 50%. Less than 4% of patients had platelet counts less than 50,000/mm3 and less than 1% had counts less than 20,000. The authors concluded that the incidence of thrombocytopenia was low and not significantly different between linezolid and vancomycin. When severe thrombocytopenia develops the best plan is to avoid both antibiotics.

Anti-platelet antigen and function tests are highly repeatable and the sensitivities are nearly 100%. Transfusing platelets is not indicated in a patient with platelet count of 55 unless there is active bleeding. Amiodarone use is not associated with thrombocytopenia.

182
Q

Postoperative cardiothoracic patients are in a highly catabolic state and they frequently have robust protein requirements of what amount per day?

A

1.5–2 g/(kg x day) of ideal body weight.

In a consensus statement a national panel recommended prealbumin as the serum marker of choice for all patients admitted to the hospital with malnutrition or nutritional risk factors such as advanced age and diabetes. They also recommended that patients with levels less than 15 mg/dL receive a formal nutrition consultation.

183
Q

Upon arrival in the ICU a man who underwent 3-vessel CABG had a BP of 110/65, heart rate of 85, C.I. of 2.0 and a PCW of 12. The grafts were judged satisfactory and postoperative EF was 35% versus 30% preoperative. One hour later he remains intubated, his core temperature is 34.5oC, and he is shivering. Bleeding is minimal and urine output for the first hour was 0.5 cc/kg.

Hypothermia is common after cardiac surgery and can lead to several derangements. What are they and how is this managed?

A

Common concerns with hypothermia include bleeding, shivering, vasoconstriction and increased afterload, decreased cardiac output, acid-base imbalance, and altered function of virtually every organ system. Since this patent is not bleeding, and cardiac index is not below 2.0, a gentle approach to correction is feasible. Intravascular volume status is probably a bit low for an elderly man with a reduced EF, but severe volume contraction is not evident. The patient’s heart rate is normal, and active, immediate measures to slow it are not advised. A NTG drip may facilitate gradual rewarming, but will compound shivering and decrease preload in the short term. Along the same line of thinking, despite marginal urine output, diuresis is not indicated. This patient will need additional volume as he rewarms, as vasodilation will follow as temperature rises.

The most pressing current issue is to stop the shivering, maintain comfort and to allow and assist rewarming. Shivering constitutes an increased metabolic load that generates heat, but it also generates CO2 at a time when elimination by breathing is difficult. This is not a preferred method to expedite rewarming. A heating blanket below and convective warming above are conventional tools for this. Intravascular volume will need to be augmented to maintain preload, but once core temperature reaches 36.5oC, his shivering will cease. Muscle relaxants can be stopped, sedation diminished and ventilator weaning can be considered. Hemodynamic parameters should improve as SVR drops and urine output will increase in response to increased cardiac index.